Методом интервалов решите неравенство: Решение Неравенств через Метод Интервалов

Содержание

Решение Неравенств через Метод Интервалов

Определение квадратного неравенства

Неравенство — алгебраическое выражение, в котором используются знаки ≠, <, >, ≤, ≥.

Числовое неравенство — это такое неравенство, в записи которого по обе стороны от знака находятся числа или числовые выражения.

Решение — значение переменной, при котором неравенство становится верным.

Решить неравенство значит найти множество, для которых оно выполняется.

Квадратное неравенство выглядит так:


где x — переменная,

a, b, c — числа,

при этом а ≠ 0.

Квадратное неравенство можно решить двумя способами:

  • графический метод;
  • метод интервалов.

Решение неравенства графическим методом

При решении квадратного неравенства необходимо найти корни соответствующего квадратного уравнения ax^2 + bx + c = 0. 2 + bx + c меньше нуля — это числовой промежуток, где парабола лежит ниже оси ОХ.

Если квадратное неравенство нестрогое, то корни входят в числовой промежуток. А если строгое — не входят.


Решение неравенства методом интервалов

Метод интервалов — это специальный алгоритм, который предназначен для решения рациональных неравенств.

Рациональное неравенство имеет вид f(x) ≤ 0, где f(x) — рациональная функция. При этом знак может быть любым: >, <, ≤, ≥.

Сейчас мы узнаем про интервалы в контексте решения квадратных неравенств.

Интервал — это некий промежуток числовой прямой, то есть все возможные числа, заключенные между двумя числами — концами интервала. Представить эти промежутки не так просто, поэтому интервалы принято рисовать.

Алгоритм решения квадратных неравенств методом интервалов:


  1. Найти нули квадратного трехчлена ax^2 + bx + c из левой части квадратного неравенства.

  2. Изобразить координатную прямую и при наличии корней отметить их на ней.

    Если неравенство строгое, нужно отметить корни пустыми (выколотыми) точками. Если нестрогое — обычными точками. Именно эти точки разбивают координатную ось на промежутки.



  3. Определить, какие знаки имеют значения трехчлена на каждом промежутке (если на первом шаге нашли нули) или на всей числовой прямой (если нулей нет). И проставить над этими промежутками + или − в соответствии с определенными знаками.

  4. Если квадратное неравенство со знаком > или ≥ — наносим штриховку над промежутками со знаками +.

    Если неравенство со знаком < или ≤, то наносим штриховку над промежутками со знаком −.

    В результате получаем геометрический образ некоторого числового множества — это и есть решение неравенства.

    Либо вместо штриховки можно нарисовать «арки» для интервалов. Справа налево, начиная с +, проставить чередуя знаки + и −.



  5. Выбрать необходимые интервалы и записать ответ. 2 есть отрицательное число -4, и свободный член -7 тоже отрицателен.

    • Когда квадратный трехчлен при D > 0 имеет два корня, то знаки его значений на промежутках чередуются. Это значит, что достаточно определить знак на одном из трех промежутков и расставить знаки над оставшимися промежутками, чередуя их. В результате возможна одна из двух последовательностей: +, −, + или −, +, −.
    • Если квадратный трехчлен при D = 0 имеет один корень, то этот корень разбивает числовую ось на два промежутка, а знаки над ними будут одинаковыми. Это значит, что достаточно определить знак над одним из них и над другим поставить такой же. При этом получится, либо +, +, либо −, −.
    • Когда квадратный трехчлен корней не имеет (D < 0), то знаки его значений на всей числовой прямой совпадают как со знаком старшего коэффициента a, так и со знаком свободного члена c.

    Теперь мы знаем пошаговый алгоритм. 2 — 5x + 6 ≥ 0.

    Как решаем:


    1. Разложим квадратный трехчлен на множители.

      Неравенство примет вид:

      (х — 3) * (х — 2) ≥ 0


    2. Проанализируем два сомножителя:

      Первый: х — 3. Этот сомножитель может поменять знак при х = 3, значит при х < 0 это выражение отрицательно: х — 3 < 0, а при х > 0 принимает положительные значения: х — 3 > 0.

      Второй: х — 2. Для этого сомножителя такая «знаковая» точка: х = 2.

      Вывод: знак произведения (х — 3) * (х — 2) меняется только при переходе переменной через значения х = 3 и х = 2.

      В этом весь смысл метода интервалов: определить интервалы значений переменной, на которых ситуация не меняется и рассматривать их как единое целое.


    3. Построим чертеж.

    4. Рассмотрим интервалы в том же порядке, как пишем и читаем: слева направо.

      х < 0 — на этом интервале ситуация не изменяется, значит, для того, чтобы определить ситуацию, можно взять любое значение из этого интервала и подставить его в произведение. Например: х = -1. Подставляем:

      (-1 — 3) * (-1 — 2) = -4 * (-3) = 12

      12 > 0

      Вывод: при х < 0 верно неравенство (х — 3) * (х — 2) > 0.

      Отобразим эти данные на чертеже:


      2 < x < 3 — на этом интервале ситуация не меняется, значит, для того, чтобы определить ситуацию нужно взять любое значение из этого интервала и подставить его в произведение. Например: х = 2,5.

      Подставляем:

      • (2,5 — 3) (2,5 — 2) = -0,5 * 0,5 = — 0,25 < 0

      Вывод: при 2 < x < 3 верно неравенство (х — 3) * (х — 2) < 0. Отметим на чертеже:


      х > 3 — на этом интервале ситуация не изменяется. Значит нужно взять любое значение из этого интервала и подставить его в произведение. Например: х = 25.

      Подставляем:

      • (25 — 3) (25 — 2) = 22*23 = 506 > 0

      Вывод: при х > 3 верно неравенство (х — 3) * (х — 2) > 0. Внесем эти данные в чертеж.


    5. Исходное неравенство: (х — 3) * (х — 2) ≥ 0.

      Если (х — 3) * (х — 2) > 0:

      (x — 3) * (x + 3/2) > 0.

      Если (х — 3) (х — 2) = 0 — при х1 = 3, х2 = 2.

      Удовлетворяющие неравенству точки закрасим, а не удовлетворяющие — оставим пустыми.


    Ответ: х ≤ 0, х ≥ 3.

    Пример 2. Применить метод интервалов для решения неравенства х2+4х+3 < 0.

    Как решить неравенство методом интервалов нам уже известно. Поэтому можем оформить решение кратко:


    Ответ: -3 < x < -2.

    Пример 3. Выполнить решение квадратного неравенства методом интервалов:


    Как решаем:


    1. Находим корни квадратного трехчлена, который находится в левой части:

    2. Так как мы решаем строгое неравенство, то на координатной прямой изображаем выколотую точку с координатой 7:

    3. Теперь определим знаки на двух полученных промежутках (−∞, 7) и (7, +∞).

      Это легко сделать, потому что дискриминант квадратного трехчлена равен нулю, а старший коэффициент со знаком минус. Фиксируем знаки: −, −:


    4. Так как мы решаем неравенство со знаком <, то изобразим штриховку над интервалами со знаками минус:

      Очевидно, решениями являются оба промежутка (−∞, 7), (7, +∞).


    5.  

    Ответ: (−∞, 7), (7, +∞).

    Метод интервалов

     

    Метод интервалов — это специальный алгоритм, предназначенный для решения сложных неравенств вида f(x) > 0. Алгоритм состоит из 5 шагов:

     

    1. Решить уравнение f(x) = 0. Таким образом, вместо неравенства получаем уравнение, которое решается намного проще;
    2. Отметить все полученные корни на координатной прямой. Таким образом, прямая разделится на несколько интервалов;
    3. Найти кратность корней. Если корни четной кратности, то над корнем рисуем петлю. (Корень считается кратным, если существует четное количество одинаковых решений)
    4. Выяснить знак (плюс или минус) функции f(x) на самом правом интервале. Для этого достаточно подставить в f(x) любое число, которое будет правее всех отмеченных корней;
    5. Отметить знаки на остальных интервалах, чередуя их.

    После этого останется лишь выписать интервалы, которые нас интересуют. Они отмечены знаком «+», если неравенство имело вид f(x) > 0, или знаком «−», если неравенство имеет вид f(x) < 0.

    В случае с нестрогими неравенствами( ≤ , ≥) необходимо включить в интервалы точки, которые являются решением уравнения f(x) = 0;

     

    Пример 1:

     

    Решить неравенство:

    (x — 2)(x + 7) < 0

    Работаем по методу интервалов.

    Шаг 1: заменяем неравенство уравнением и решаем его:

    (x — 2)(x + 7) = 0

    Произведение равно нулю тогда и только тогда, когда хотя бы один из множителей равен нулю:

    x — 2 = 0 => x = 2

    x + 7 = 0 => x = -7

    Получили два корня.

     

    Шаг 2:

     отмечаем эти корни на координатной прямой. Имеем:

     

     

    Шаг 3: находим знак функции на самом правом интервале (правее отмеченной точки x = 2). Для этого надо взять любое число, которое больше числа x = 2. Например, возьмем x = 3 (но никто не запрещает взять x = 4, x = 10 и даже x = 10 000). 

    Получим:

    f(x) = (x — 2)(x + 7)

    x = 3

    f(3)=(3 — 2)(3 + 7) = 1*10 = 10

    Получаем, что f(3) = 10 > 0 (10 – это положительное число), поэтому в самом правом интервале ставим знак плюс.

     

    Шаг 4:  нужно отметить знаки на остальных интервалах. Помним, что при переходе через каждый корень знак должен меняться. Например, справа от корня x = 2 стоит плюс (мы убедились в этом на предыдущем шаге), поэтому слева обязан стоять минус. Этот минус распространяется на весь интервал (−7; 2), поэтому справа от корня x = −7 стоит минус. Следовательно, слева от корня x = −7 стоит плюс. Осталось отметить эти знаки на координатной оси. 

     

     

    Вернемся к исходному неравенству, которое имело вид:

    (x — 2)(x + 7) < 0

    Итак, функция должна быть меньше нуля. Значит, нас интересует знак минус, который возникает лишь на одном интервале: (−7; 2). Это и будет ответ.

     

    Пример 2:

     

    Решить неравенство:

    (9x— 6x + 1)(x — 2) ≥ 0

    Решение: 

    Для начала необходимо найти корни уравнения 

    (9x— 6x + 1)(x — 2) = 0

    Свернем первую скобку, получим:

    (3x — 1)2(x — 2) = 0

    Отсюда:

    x — 2 = 0; (3x — 1)2 = 0

    Решив эти уравнения получим:

    x= 2; x

    2 = ; x3= ;

    Нанесем точки на числовую прямую:

    Т.к. xи x– кратные корни, то на прямой будет одна точка и над ней “петля”.

    Возьмем любое число меньшее самой левой точки   и подставим в исходное неравенство. Возьмем число -1.

    (9*(-1)— 6*(-1) + 1)(-1 — 2) = -12

    Т.к. решение уравнения при x = -1 отрицательное (-12), то на графике в крайнем левом интервале пишем -, и далее чередуя знак записываем его в следующие интервалы:

    Далее выбираем отрицательные интервалы, т. к. знак нашего неравенства ≤.

    Не забываем включать решение уравнения (найденные X), т.к. наше неравенство нестрогое.

    Ответ: {} U [2;+∞)

     

    Пример 3:

     

    Решить неравенство:

    (9x

    — 6x + 1)(x — 2) > 0

    Все, чем данное неравенство отличается от предыдущего – вместо нестрогого неравенства (≥) стоит строгое (>). Как ни странно, решение данного неравенства будет иным.

    Найдем корни уравнения (9x— 6x + 1)(x — 2) ≠ 0 (знак ≠ означает, что найденные корни не могут быть решениями нашего неравенства, т.к. оно строгое). Проделав все этапы, что и в предыдущем примере получим:

    x1= 2; x2,3 =;

    Вынесем наши решения на числовую прямую (обратите внимания, что данные точки не включены, т.к. неравенство строгое, т.е. левая часть неравенства не равна нулю)

    Обратите внимание, что корни x2 и x3 совпадают, корень “” является кратным. Соответственно, в данной точке на числовой прямой рисуем петлю.

    Возьмем число -1.

    (9*(-1)— 6*(-1) + 1)(-1 — 2) = -12

    Т.к. решение уравнения при x = -1 отрицательное (-12), то на графике в крайнем левом интервале пишем -, и далее чередуя знак записываем его в следующие интервалы:

    Далее выбираем отрицательные интервалы, т.к. знак нашего неравенства <.

    Найденные корни не включаем в ответ.

    Ответ: (2;+∞).

    Неравенства методом интервалов

    Рассмотрим, как решать неравенства методом интервалов, на конкретных примерах.

       

    Используем алгоритм метода интервалов. Приравниваем к нулю левую часть:

       

       

       

    Полученные точки отмечаем на числовой прямой:

    Для проверки знака берем 0 (желательно на числовой прямой отметить взятую точку, чтобы потом не забыть, куда ставить знак). Подставляем 0 в последнее неравенство: (2∙0-14)(5∙0+25)= -14∙25, то есть (-)∙(+)= -. Таким образом, в промежуток, из которого взяли нуль, ставим знак «-«, остальные знаки чередуем в шахматном порядке. Поскольку решаем неравенство ≥0, выбираем промежутки со знаком «+» и записываем ответ.

    Ответ:

       

       

    Приравниваем к нулю левую часть:

       

       

    Полученные точки отмечаем на числовой прямой:

    Для проверки знака берем 0 и подставляем его в последнее неравенство. По знакам получаем:

       

    В промежуток, которому принадлежит 0, ставим «+», остальные знаки расставляем в шахматном порядке. Поскольку решаем неравенство ≤0, в ответ выбираем промежутки со знаком «-«. (Не забываем, когда точки закрашенные, а когда — выколотые. Те точки, в которых знаменатель обращается в нуль, выколотые всегда).

    Ответ:

       

       

    Приравниваем к нулю левую часть:

       

    По теореме, обратной теореме Виета

       

    Полученные точки отмечаем на числовой прямой:

    Для определения знака берем 0 и подставляем его в последнее неравенство. Получает (-)/(-)=(+). Остальные знаки расставляем в шахматном порядке. Поскольку решаем неравенство ≥0, выбираем промежутки со знаком «+» и записываем ответ.

    Ответ:

       

       

    Переносим все слагаемые в левую часть, приводим к наименьшему общему знаменателю и упрощаем: 

       

       

       

       

    После упрощения решаем неравенство методом интервалов.

    Приравниваем к нулю левую часть:

       

       

    Точек, в которых числитель обращается в нуль, нет. На числовой прямой отмечаем только одну точку:

    Для проверки берем нуль. Подставляя его в последнее неравенство, получаем «+». На другом интервале — «-«. Нам нужен интервал с «-«.

    Ответ:

       

    Как решать более сложные неравенства методом интервалов, рассмотрим в следующий раз.

    Подготовка школьников к ЕГЭ и ОГЭ (Справочник по математике — Алгебра

          Определение. Рациональным неравенством называют такое неравенство, которое при помощи равносильных преобразований сводится к одному из следующих неравенств

    где   P(x)   и   Q(x)   – многочлены.

         Для решения рациональных неравенств часто используют удобный способ, который получил название «метод интервалов». Продемонстрируем применение метода интервалов на примерах.

          Пример 1. Решить неравенство

    (1)

          Решение. Вводя обозначение

    (2)

    перепишем неравенство (1) в виде

    (3)

          Заметим, что и числитель, и знаменатель дроби из правой части формулы (2) являются произведением выражений типа

    где   a   – вещественное число, а   k   – натуральное число. Действительно, произведение выражений

    (5)

    равно числителю дроби (2), а произведение выражений

    (6)

    равно знаменателю дроби (2).

          В случае, когда показатель степени k   в формуле (4) является нечётным числом, выражение (4) отрицательно для всех значений   x,   лежащих на числовой оси слева от числа   a,   и положительно для всех значений   x,   лежащих на числовой оси справа от числа   a.

          В случае, когда показатель степени k   в формуле (4) является чётным числом, выражение (4) положительно для всех значений   x,   отличных от числа   a.

          Следовательно, если отметить на числовой оси числа

    (7)

    играющие роль числа   a   из формулы (4) в выражениях (5), а также отметить на числовой оси числа

    (8)

    играющие роль числа   a   из формулы (4) в выражениях (6), то внутри каждого из промежутков, полученных на числовой оси, функция (2) будет сохранять свой знак. Другими словами, внутри каждого из полученных промежутков числовой оси значения функции (2) будут или положительными, или отрицательными.

          Для того, чтобы воспользоваться этим рассуждением, отметим на числовой оси в нужном порядке числа (7) и (8), причём числа (7) изобразим закрашенными кружками, а числа (8) – незакрашенными кружками (рис.1).

          Перейдём от рисунка 1 к рисунку 2.

          На рисунке 2 подчёркнуты числа   3   и   4,   то есть те из чисел (7) и (8), которым соответствуют чётные показатели степени в выражениях (5) и (6). Действительно, числам   3   и   4   соответствуют чётные показатели степени в выражениях (5) и (6)подчёркнуто число   3,   то есть то из чисел (7) и (8), которому соответствует чётный показатель степени в выражениях (5) и (6). Действительно, числу   3   соответствует чётный показатель степени в выражении (6), поскольку числу   3   соответствует выражение

    (x – 3 )100

    с показателем степени   100,   а числу   4   соответствует выражение.

    (x – 4 )2

    с показателем степени   2 .

          Нанесем на рисунок 2 волновую линию, начиная от правого верхнего угла рисунка и двигаясь влево (рис.3) .

          Мы начинаем вести волновую линию от правого верхнего угла рисунка, поскольку справа от всех точек (7) и (8) функция (2) принимает положительные значения.

          Заметим, что волновая линия на рисунке (3) вблизи от подчёркнутых точек 3 и 4подчёркнутой точки 3 располагается по одну сторону от числовой оси, а в точках

    (9)

    которым соответствуют нечётные показатели степени в выражениях (5) и (6), пересекает числовую ось и вблизи от этих точек располагается по разные стороны от числовой оси.

          Важно отметить, что значения функции (2) внутри каждого из промежутков имеют один и тот же знак. При переходе от промежутка к соседнему промежутку через точки (9), которым соответствуют нечётные показатели степени в выражениях (5) и (6), значения функции (2) меняют знак на противоположный. При переходе от промежутка к соседнему промежутку через точки   3   и   4,   которым соответствуют чётные показателиточку   3,   которой соответствует чётный показатель степени в выражениях (5) и (6), значения функции (2) знак не изменяют.

          Нанесем на рисунок 3 знаки   « + »   и   « – »,   как показано на рисунке 4.

          На промежутках, отмеченных знаком   « + »,   функция (2) принимает положительные значения. На промежутках, отмеченных знаком   « – »,   функция (2) принимает отрицательные значения. Отсюда вытекает, что решением неравенства (1) является объединение промежутков, отмеченных знаком   « – »,   поскольку именно на этих промежутках функция (2) принимает отрицательные значения. Для завершения решения примера остаётся лишь добавить, что концы промежутков

    отмеченные на рисунках закрашенными кружками, входят в ответ задачи, а концы промежутков

    отмеченные на рисунках незакрашенными кружками, не входят в ответ задачи.

    Ответ:

          Пример 2. Решить неравенство

    (10)

          Решение. Преобразуем неравенство (10) к такому виду, чтобы можно было применить метод интервалов:

          Неравенство

    (11)

    имеет вид (1). Решим его методом интервалов. Для этого отметим незакрашенными кружками числа

          Проведём волну, начиная движение от правого верхнего угла, и отметим знаками   « + »   и   « – »   промежутки числовой оси (рис.6)

          Решением неравенства (11) являются промежутки, отмеченные знаком   « – » .   Концы промежутков в ответ не входят. 

          Ответ:

          Замечание. Рекомендуем ознакомиться с нашим учебным пособием «Решение рациональных неравенств», близко связанным с материалом данного раздела справочника.

          На сайте можно также ознакомиться с нашими учебными материалами для подготовки к ЕГЭ и ОГЭ по математике.

    § Как решать квадратные неравенства. Метод интервалов

    Требуется решить квадратное неравенство.

    x2 + x − 12

    Итак, согласно п.1 мы должны перенести все члены неравенства в левую часть, так чтобы в правой остался только ноль. В заданном неравенстве «x2 + x − 12 » ничего дополнительно делать не требуется, так как в правой части и так уже стоит ноль.

    Переходим к п.2. Необходимо сделать так, чтобы перед «x2» стоял положительный коэффициент. В неравенстве «x2 + x − 12 » при «x2» стоит положительный коэффициент «1», значит, снова нам ничего делать не требуется.

    Согласно п.3 приравняем левую часть неравенства к нулю и решим полученное квадратное уравнение.

    x2 + x − 12 = 0

    x1;2 =

    −1 ± √12 − 4 · 1 · (−12)
    2 · 1

    x1;2 =

    x1;2 =

    x1;2 =


    x1 = x2 =
    x1 = x2 =
    x1 = −4 x2 = 3

    Теперь по п.4 отметим полученные корни на числовой оси в порядке возрастания.

    Помните, что, исходя их того, какое перед нами неравенство (строгое или нестрогое) мы отмечаем точки на числовой оси разным образом.

    Теперь, как сказано в п. 5, нарисуем «арки» над интервалами между отмеченными точками.

    Проставим знаки внутри интервалов. Справа налево чередуя, начиная с «+», отметим знаки.

    Нам осталось только выполнить пункт 6, то есть выбрать нужные интервалы и записать их в ответ. Вернемся к нашему неравенству.

    Так как в нашем неравенстве «x2 + x − 12 », значит, нам требуются отрицательные интервалы. Заштрихуем все отрицательные области на числовой оси и выпишем их в ответ.

    Отрицательным интервалом оказался лишь один, который находится между числами «−4» и «3», поэтому запишем его в ответ в виде двойного неравенства
    −4 .

    Запишем полученный ответ квадратного неравенства.

    Ответ: −4

    Именно из-за того, что при решении квадратного неравенства мы рассматриваем интервалы между числами, метод интервалов и получил свое название.

    После получения ответа имеет смысл сделать его проверку, чтобы убедиться в правильности решения.

    Выберем любое число, которое находится в заштрихованной области полученного ответа −4 и подставим его вместо «x» в исходное неравенство. Если мы получим верное неравенство, значит мы нашли ответ квадратного неравенства верно.

    Возьмем, например, из интервала число «0». Подставим его в исходное неравенство «x2 + x − 12 ».

    x2 + x − 12

    02 + 0 − 12 −12 (верно)

    Мы получили верное неравенство при подстановке числа из области решений, значит ответ найден правильно.

    Краткая запись решения методом интервалов

    Сокращенно запись решения квадратного неравенства «x2 + x − 12 » методом интервалов будет выглядеть так:

    x2 + x − 12

    x2 + x − 12 = 0

    x1;2 =

    −1 ± √12 − 4 · 1 · (−12)
    2 · 1

    x1;2 =

    x1;2 =

    x1;2 =

    x1 = x2 =
    x1 = x2 =
    x1 = −4 x2 = 3
    Ответ: −4

    Другие примеры решения квадратных неравенств

    Рассмотрим решение других примеров квадратных неравенств. Требуется решить квадратное неравенство:

    2x2 − x ≥ 0

    В правой части неравенство уже стоит ноль. При «x2» стоит «2» (положительный коэффициент), значит можно сразу переходить к поиску корней.

    2x2 − x ≥ 0

    2x2 − x = 0

    x1;2 =

    −(−1) ± √(−12) − 4 · 2 · 0
    2 · 2

    x1;2 =

    x1;2 =

    x1 = x2 =
    x1 = x2 =
    x1 = x2 = 0
    Ответ: x ≤ 0;    x ≥

    Рассмотрим пример, где перед «x2» в квадратном неравенстве стоит отрицательный коэффициент.

    −x2 − 3x + 4 ≥ 0

    По п.2 общих правил решения методом интервалов нам нужно сделать так, чтобы перед «x2» стоял положительный коэффициент. Для этого умножим все неравенство на «−1».

                −x2 − 3x + 4 ≥ 0 | ·(−1)
    x2 + 3x − 4 ≤ 0

    Можно переходить к п.4 и п.5. Приравняем левую часть неравенства к нулю и решим полученное квадратное уравнение. Затем расположим полученные корни на числовой оси и проведем между ними «арки».

    x2 + 3x − 4 ≤ 0

    x2 + 3x − 4 = 0

    x1;2 =

    −3 ± √32 − 4 · 1 · (−4)
    2 · 1

    x1;2 =

    x1;2 =

    x1;2 =

    x1 = x2 =
    x1 = x2 =
    x1 = 4 x2 = −1
    Важно!

    При определении того какие интервалы нам нужно брать в ответ, исходить нужно из самого последнего изменения неравенства перед нахождением его корней.

    В нашем случае самая последняя версия неравенства перед поиском корней уравнения это «x2 + 3x − 4 ≤ 0».

    Значит для ответа нужно выбирать интервалы со знаком «−».

    Ответ: −1 ≤ x ≤ 4

    К сожалению, при решении квадратного неравенства не всегда получаются два корня и все идет по общему плану выше. Возможны случаи, когда получается один корень или даже ни одного корня.

    Как решить квадратные неравенства в таких случаях, мы разберем в следующем уроке «Квадратные неравенства с одним корнем или без корней».



    Решение неравенств методом интервалов тест по алгебре

    Сложность: знаток.Последний раз тест пройден более 24 часов назад.

    Материал подготовлен совместно с учителем высшей категории

    Опыт работы учителем математики — более 33 лет.

    1. Вопрос 1 из 10

      Решите неравенство

      • Правильный ответ
      • Неправильный ответ
      • Вы и еще 61% ответили правильно
      • 61% ответили правильно на этот вопрос

      В вопросе ошибка?

      Следующий вопросОтветить
    2. Вопрос 2 из 10

      Решением неравенства x2 + 2x – 48 < 0 является промежуток

      • Правильный ответ
      • Неправильный ответ
      • Вы и еще 72% ответили правильно
      • 72% ответили правильно на этот вопрос

      В вопросе ошибка?

      Ответить
    3. Вопрос 3 из 10

      Решите неравенство

      • Правильный ответ
      • Неправильный ответ
      • Вы и еще 67% ответили правильно
      • 67% ответили правильно на этот вопрос

      В вопросе ошибка?

      Ответить
    4. Вопрос 4 из 10

      Решением неравенства (х – 2)(х – 5)(х – 4) > 0 является промежуток

      • Правильный ответ
      • Неправильный ответ
      • Вы и еще 73% ответили правильно
      • 73% ответили правильно на этот вопрос

      В вопросе ошибка?

      Ответить
    5. Вопрос 5 из 10

      Решите неравенство

      • Правильный ответ
      • Неправильный ответ
      • Вы и еще 56% ответили правильно
      • 56% ответили правильно на этот вопрос

      В вопросе ошибка?

      Ответить
    6. Вопрос 6 из 10

      Решением неравенства (x — 3)(x — 9) > 0 является промежуток

      • Правильный ответ
      • Неправильный ответ
      • Вы и еще 60% ответили правильно
      • 60% ответили правильно на этот вопрос

      В вопросе ошибка?

      Ответить
    7. Вопрос 7 из 10

      Решите неравенство

      • Правильный ответ
      • Неправильный ответ
      • Вы и еще 56% ответили правильно
      • 56% ответили правильно на этот вопрос

      В вопросе ошибка?

      Ответить
    8. Вопрос 8 из 10

      Какой промежуток является решением неравенства (2 + x)(14 — x) < 0?

      • Правильный ответ
      • Неправильный ответ
      • Вы и еще 62% ответили правильно
      • 62% ответили правильно на этот вопрос

      В вопросе ошибка?

      Ответить
    9. Вопрос 9 из 10

      Решите неравенство

      • Правильный ответ
      • Неправильный ответ
      • Вы и еще 53% ответили правильно
      • 53% ответили правильно на этот вопрос

      В вопросе ошибка?

      Ответить
    10. Вопрос 10 из 10

      Какой промежуток является решением неравенства x(8 — x)(12 + x) > 0?

      • Правильный ответ
      • Неправильный ответ
      • Вы и еще 65% ответили правильно
      • 65% ответили правильно на этот вопрос

      В вопросе ошибка?

      Ответить

    Доска почёта

    Чтобы попасть сюда — пройдите тест.

    ТОП-4 тестакоторые проходят вместе с этим
    Рейтинг теста

    Средняя оценка: 3.7. Всего получено оценок: 470.

    А какую оценку получите вы? Чтобы узнать — пройдите тест.

    Решение неравенств второй степени с одной переменной. Метод интервалов 9 класс онлайн-подготовка на Ростелеком Лицей |

    Тема 6.

    Решение неравенств второй степени с одной переменной. Метод интервалов.

    Неравенства вида

    ax2 + bx + c > 0 и ax2 + bx + c

    где x – переменная, a, b и c – некоторые числа и a ≠ 0, называют неравенствами второй степени с одной переменной.

    Решение неравенства ax2 + bx + c > 0 или ax2 + bx + c y = ax2 + bx + c принимает положительные или отрицательные значения. Для этого достаточно проанализировать, как расположен график функции y = ax2 + bx + c в координатной плоскости: куда направлены ветви параболы – вверх или вниз, пересекает ли парабола ось x и если пересекает, то в каких точках.

    Пример:

    Решить неравенство: x2 + 2x — 48

    Введем функцию y = x2 + 2x — 48.

    Графиком этой функции является парабола, ветви которой направлены вверх, так как a = 1.

    Выясним, как расположен график этой функции относительно оси x. Для этого решим квадратное уравнение x2 + 2x — 48 = 0.

    Это уравнение имеет два корня:

    x1 = -8 и x2 = 6.

    Значит, парабола y = x2 + 2x — 48 пересекает ось x в двух точках, абсциссы которых равны -8 и 6. Схематично изобразим эту параболу.

    Ответ: x∈-8;6

    Решим неравенство:

    x2 + 2x + 15

    График функции y = —x2 + 2x + 15 – это парабола, ветви которой направлены вниз, так как a

    Выясним, как расположен график функции y = —x2 + 2x + 15 в координатной плоскости, пересекает ли он ось x и в каких точках.

    Для этого решим уравнение:

    x2 + 2x + 15 = 0

    x1=-3; x2=5

    Схематично изобразим эту параболу

    Функция принимает отрицательные значения при x принадлежит -∞;-3 или 5;+∞.

    Ответ: x∈-∞;-3∪5;+∞

    Решим неравенство:

    2x2 — 3x + 8 > 0

    Графиком функции y = 2x2 — 3x + 8 является парабола, ветви которой направлены вверх, так как a > 0. Выясним, как располагается эта парабола относительно оси x. Для этого решим квадратное уравнение:

    2x2 — 3x + 8 = 0

    D = 9 — 4 ∙ 2 ∙ 8 = -55

    Данное уравнение не имеет корней, значит, парабола не пересекает ось x. Схематично покажем, как располагается эта парабола относительно оси x.

    Из рисунка видно, что данная функция принимает положительные значения при любом значении x.

    Ответ: -∞;+∞

    Итак, для решения неравенств вида

    ax2 + bx + c > 0 и ax2 + bx + c

    1. Выяснить имеет ли квадратный трехчлена ax2 + bx + c имеет ли трехчлен корни;
    2. Если трехчлен имеет корни, то отмечают их на оси x и через отмеченные точки проводят схематически параболу, ветви которой направлены вверх, если a > 0 или вниз, если a a > 0 или в нижней полуплоскости при a
    3. На оси x найти промежутки, для которых точки параболы расположены выше оси x (если решают неравенство ax2 + bx + c > 0) или ниже оси x (если решают неравенство ax2 + bx + c

    Рассмотрим функцию

    fx=x+1x-2x+3

    Областью определения этой функции является множество всех чисел. Точки -3, -1 и 2 нули функции, которые разбивают область определения на промежутки -∞;-3,-3;-1,-1;2,2;+∞. Выясним знак функции в каждом из указанных промежутков.

    Выражение (x + 1)(x — 2)(x + 3) представляет собой произведение трех множителей. Знак каждого из этих множителей в рассматриваемых промежутках указан в таблице.

    -∞;-3

    -3;-1

    -1;2

    2;+∞

    x + 3

    +

    +

    +

    x + 1

    +

    +

    x — 2

    +

    Отсюда ясно, что:

    Если x∈-∞;-3, то fx<0;

    Если x∈-3;-1, то fx>0;

    Если x∈-1;2, то fx<0;

    Если x∈2;+∞, то fx>0;

    Видно, что в каждом из промежутков

    -∞;-3,-3;-1,-1;2,2;+∞ функция сохраняет знак, а при переходе через точки -3, -1 и 2 ее знак изменяется.

    Вообще пусть функция задана формулой

    fx=x-x1x-x2…x-xn, где x – переменная, а x1, x2, …, xn не равные друг другу числа. Числа x1, x2, …, xn являются нулями функции. В каждом из промежутков, на которые область определения разбивается нулями функции, знак функции сохраняется, а при переходе через ноль знак изменяется.

    Это свойство используется для решений неравенств вида:

    x-x1x-x2…x-xn>0,

    x-x1x-x2…x-xn<0.

    x-5x+3x+7>0

    Введем функцию fx=x-5x+3x+7

    Найдем нули функции: -7, -3 и 5

    Определим знак функции в каждом из этих промежутков. В крайнем правом промежутке функция положительна, а далее знаки чередуются.

    Ответ: -7;-3∪5;+∞

    Итак, чтобы решить неравенство методом интервалов, надо:

    1. Ввести функцию;
    2. Найти нули этой функции;
    3. Нанести нули функции на числовую прямую;
    4. Определить знак в каждом промежутке;
    5. Посмотреть знак и выделить нужный интервал.

    Решить неравенства | Начальная алгебра

    Цели обучения

    • Опишите решения проблемы неравенства
      • Изобразите неравенства на числовой прямой
      • Изобразите неравенства, используя обозначение интервалов
    • Решите пошаговые неравенства
      • Используйте свойства сложения и умножения, чтобы решать алгебраические неравенства и выражать их решения графически и с интервальной нотацией
      • Решите неравенства, содержащие абсолютное значение
    • Решите многоступенчатые неравенства
      • Объединение свойств неравенства для выделения переменных, решения алгебраических неравенств и графического представления их решений
      • Упростите и решите алгебраические неравенства, используя свойство распределенности для удаления скобок и дробей

    Изобразите неравенства на числовой прямой

    Во-первых, давайте определимся с важной терминологией. Неравенство — это математическое утверждение, которое сравнивает два выражения, используя идеи больше или меньше чем. В этих утверждениях используются специальные символы. Когда вы читаете неравенство, читайте его слева направо — как если бы вы читали текст на странице. В алгебре неравенства используются для описания больших наборов решений. Иногда существует бесконечное количество чисел, которые удовлетворяют неравенству, поэтому вместо того, чтобы пытаться перечислить бесконечное количество чисел, мы разработали несколько способов краткого описания очень больших списков.

    Первый способ, с которым вы, вероятно, знакомы — основное неравенство. Например:

    • [latex] {x} \ lt {9} [/ latex] указывает список чисел, которые меньше 9. Вы бы предпочли написать [latex] {x} \ lt {9} [/ latex] или попробовать перечислить все возможные числа меньше 9? (надеюсь, ваш ответ отрицательный)
    • [латекс] -5 \ le {t} [/ latex] указывает все числа, которые больше или равны [latex] -5 [/ latex].

    Обратите внимание, как размещение переменной слева или справа от знака неравенства может изменить, ищете ли вы больше или меньше.

    Например:

    • [латекс] x \ lt5 [/ latex] означает все действительные числа, которые меньше 5, тогда как;
    • [latex] 5 \ lt {x} [/ latex] означает, что 5 меньше x, или мы могли бы переписать это с x слева: [latex] x \ gt {5} [/ latex] обратите внимание, как неравенство по-прежнему указывает то же направление относительно x. Этот оператор представляет все действительные числа, которые больше 5, что легче интерпретировать, чем 5 меньше x.

    Второй способ — построить график с использованием числовой прямой:

    И третий способ — с интервалом.

    В этом разделе мы подробно рассмотрим второй и третий способы. Опять же, эти три способа написать решения неравенства:

    • неравенство
    • интервал
    • график

    Признаки неравенства

    В рамке ниже показаны символ, значение и пример для каждого знака неравенства. Иногда легко запутаться в неравенствах, просто не забывайте читать их слева направо.

    Символ слов Пример
    [латекс] \ neq [/ латекс] не равно [латекс] {2} \ neq {8} [/ latex], 2 это не равно до 8 .
    [латекс] \ gt [/ латекс] больше [латекс] {5} \ gt {1} [/ латекс], 5 больше, чем 1
    [латекс] \ lt [/ латекс] менее [латекс] {2} \ lt {11} [/ латекс], 2 меньше 11
    [латекс] \ geq [/ латекс] больше или равно [латекс] {4} \ geq {4} [/ latex], 4 больше или равно 4
    [латекс] \ leq [/ латекс] меньше или равно [латекс] {7} \ leq {9} [/ latex], 7 меньше или равно 9

    Неравенство [latex] x> y [/ latex] также можно записать как [latex] {y} <{x} [/ latex]. Стороны любого неравенства можно поменять местами, если символ неравенства между ними также перевернут.

    Графическое изображение неравенства

    Неравенства также можно изобразить на числовой прямой. Ниже приведены три примера неравенств и их графики. Графики — очень полезный способ визуализировать информацию, особенно когда эта информация представляет собой бесконечный список чисел!

    [латекс] х \ leq -4 [/ латекс]. Это переводится во все действительные числа в числовой строке, которые меньше или равны 4.

    [латекс] {x} \ geq {-3} [/ латекс]. Это переводится во все действительные числа в числовой строке, которые больше или равны -3.

    Каждый из этих графиков начинается с круга — открытого или замкнутого (заштрихованного) круга. Эту точку часто называют конечной точкой решения. Замкнутый или заштрихованный круг используется для обозначения неравенств больше или равно [latex] \ displaystyle \ left (\ geq \ right) [/ latex] или меньше или равно [latex] \ displaystyle. \ left (\ leq \ right) [/ латекс].Дело в том, что это часть решения. Открытый кружок используется для больше (>) или меньше (<). Дело в , а не в части решения.

    Затем график бесконечно продолжается в одном направлении. Это показано линией со стрелкой в ​​конце. Например, обратите внимание, что для графа [latex] \ displaystyle x \ geq -3 [/ latex], показанного выше, конечной точкой является [latex] −3 [/ latex], представленная замкнутым кругом, поскольку неравенство составляет больше или равно [латекс] -3 [/ латекс].Синяя линия рисуется справа от числовой, потому что значения в этой области больше, чем [latex] −3 [/ latex]. Стрелка в конце указывает, что решения продолжаются бесконечно.

    Пример

    График неравенства [латекс] x \ ge 4 [/ латекс]

    Показать решение

    Мы можем использовать числовую линию, как показано. Поскольку значения x включают 4, мы помещаем сплошную точку на числовой прямой с номером 4.

    Затем мы рисуем линию, которая начинается с [latex] x = 4 [/ latex] и, как указано стрелкой, продолжается до положительной бесконечности, что показывает, что набор решений включает все действительные числа, большие или равные 4.

    В этом видео показан пример построения графика неравенства.

    Пример

    Напишите неравенство, описывающее все действительные числа на числовой прямой, которые меньше 2, а затем нарисуйте соответствующий график.

    Показать решение

    Нам нужно начать слева и работать вправо, поэтому мы начинаем с отрицательной бесконечности и заканчиваем на [latex] -2 [/ latex]. Мы не будем включать ни то, ни другое, потому что бесконечность не является числом, и неравенство не включает [латекс] -2 [/ латекс].

    Неравенство: [латекс] x <2 [/ латекс]

    Чтобы нарисовать график, сначала поместите открытую точку на числовой прямой, а затем нарисуйте линию, идущую влево. Нарисуйте стрелку в самой левой точке линии, чтобы указать, что она продолжается до бесконечности.

    В следующем видео показано, как математически написать неравенство, если оно выражается словами. Затем мы построим его график.

    Изобразите неравенства, используя обозначение интервала

    Другой широко используемый и, возможно, самый краткий метод описания неравенств и решений неравенств называется интервальной нотацией . Согласно этому соглашению, наборы состоят из круглых или квадратных скобок, каждая из которых имеет свое значение. Решения для [latex] x \ geq 4 [/ latex] представлены как [latex] \ left [4, \ infty \ right) [/ latex]. Этот метод широко используется и будет присутствовать в других курсах математики, которые вы, возможно, пройдете.

    Основная концепция, которую следует запомнить, заключается в том, что круглые скобки представляют решения больше или меньше числа, а квадратные скобки представляют решения, которые больше или равны или меньше или равны числу.Используйте круглые скобки для обозначения бесконечности или отрицательной бесконечности, поскольку положительная и отрицательная бесконечность не являются числами в обычном смысле слова и, следовательно, не могут быть «равны». Несколько примеров интервала или набора чисел, в который попадает решение: [latex] \ left [-2,6 \ right) [/ latex] или все числа между [latex] -2 [/ латекс] и [латекс] 6 [/ латекс], включая [латекс] -2 [/ латекс], но не включая [латекс] 6 [/ латекс]; [latex] \ left (-1,0 \ right) [/ latex], все действительные числа между, но не включая [latex] -1 [/ latex] и [latex] 0 [/ latex]; и [latex] \ left (- \ infty, 1 \ right] [/ latex], все действительные числа меньше, включая [latex] 1 [/ latex].В таблице ниже представлены возможные варианты. Не забывайте читать неравенства слева направо, как текст.

    В таблице ниже описаны все возможные неравенства, которые могут возникнуть, и способы их записи с использованием интервальной записи, где a и b — действительные числа.

    The entries in the third row are: All real numbers greater than a, but not including a; {x| x > a}; (a , infinity). The entries in the fourth row are: All real numbers less than b, but not including b; {x| x < b}; (negative infinity, b). The entries in the fifth row are: All real numbers greater than a, including a; {x| x a}; [a, infinity). The entries in the sixth row are: All real numbers less than b, including b; {x| x b}; (negative infinity, b]. The entries in the seventh row are: All real numbers between a and b, including a; {x| a x < b}; [a, b). The entries in the eighth row are: All real numbers between a and b, including b; {x| a < x b}; (a, b]. The entries in the ninth row are: All real numbers between a and b, including a and b; {x| a x b}; [a, b]. The entries in the tenth row are: all real numbers less than a and greater than b; {x| x < a and x > b}; (negative infinity, a) union (b, infinity). The entries in the eleventh row are: All real numbers; {x| x is all real numbers}; (negative infinity, infinity). «>
    Неравенство слов Интервальное обозначение
    [латекс] {a} \ lt {x} \ lt {b} [/ латекс] все действительные числа от a до b , не включая a и b [латекс] \ левый (а, б \ правый) [/ латекс]
    [латекс] {x} \ gt {a} [/ латекс] Все действительные числа больше a , но не включая a [латекс] \ слева (a, \ infty \ right) [/ латекс]
    [латекс] {x} \ lt {b} [/ латекс] Все действительные числа меньше b , но не включая b [латекс] \ влево (- \ infty, b \ right) [/ латекс]
    [латекс] {x} \ ge {a} [/ латекс] Все действительные числа больше a , включая a [латекс] \ left [a, \ infty \ right) [/ latex]
    [латекс] {x} \ le {b} [/ латекс] Все действительные числа меньше b , включая b [латекс] \ влево (- \ infty, b \ right] [/ латекс]
    [латекс] {a} \ le {x} \ lt {b} [/ латекс] Все действительные числа от до и b , включая и [латекс] \ слева [a, b \ справа) [/ латекс]
    [латекс] {a} \ lt {x} \ le {b} [/ латекс] Все действительные числа между a и b , включая b [латекс] \ левый (а, б \ правый] [/ латекс]
    [латекс] {a} \ le {x} \ le {b} [/ latex] Все действительные числа между a и b , включая a и b [латекс] \ слева [a, b \ справа] [/ латекс]
    [латекс] {x} \ lt {a} \ text {или} {x} \ gt {b} [/ latex] Все действительные числа меньше a или больше b [латекс] \ left (- \ infty, a \ right) \ чашка \ left (b, \ infty \ right) [/ latex]
    Все вещественные числа Все вещественные числа [латекс] \ левый (- \ infty, \ infty \ right) [/ латекс]

    Пример

    Опишите неравенство [латекс] x \ ge 4 [/ latex], используя обозначение интервала

    Показать решение

    Решения для [latex] x \ ge 4 [/ latex] представлены как [latex] \ left [4, \ infty \ right) [/ latex].

    Обратите внимание на использование кронштейна слева, потому что 4 включены в набор решений.

    В следующем видео мы показываем еще один пример использования обозначения интервалов для описания неравенства.

    Пример

    Используйте обозначение интервала, чтобы указать все действительные числа, большие или равные [latex] -2 [/ latex].

    Показать решение

    Используйте скобку слева от [latex] -2 [/ latex] и скобки после бесконечности: [latex] \ left [-2, \ infty \ right) [/ latex].Скобка указывает, что [латекс] -2 [/ латекс] включен в набор со всеми действительными числами от [латекс] -2 [/ латекс] до бесконечности.

    В следующем видео мы показываем еще один пример преобразования слов в неравенство и записи его в интервальной нотации, а также рисование графика.

    Подумай об этом

    В предыдущих примерах вам давали неравенство или описание одного со словами и просили нарисовать соответствующий график и записать интервал.В этом примере вам дается интервал и предлагается записать неравенство и нарисовать график.

    Дано [latex] \ left (- \ infty, 10 \ right) [/ latex], запишите соответствующее неравенство и нарисуйте график.

    В поле ниже запишите, считаете ли вы, что сначала будет проще нарисовать график или сначала записать неравенство.

    Показать решение

    Сначала нарисуем график.

    Интервал читается как «все действительные числа меньше 10», поэтому мы начнем с того, что поставим точку на 10 и проведем линию влево со стрелкой, указывающей, что решение продолжается до отрицательной бесконечности.

    Чтобы записать неравенство, мы будем использовать <, поскольку круглые скобки указывают, что 10 не включено. [латекс] x <10 [/ латекс]

    В следующем видео вы увидите примеры того, как нарисовать график с учетом неравенства в обозначении интервалов.

    И, наконец, последнее видео, в котором показано, как записывать неравенства с помощью графика, с обозначением интервалов и в виде неравенства.

    Решите пошаговые неравенства

    Решите неравенства сложением и вычитанием

    Вы можете решить большинство неравенств, используя обратные операции, как вы это делали для решения уравнений.Это потому, что, когда вы добавляете или вычитаете одно и то же значение с обеих сторон неравенства, вы сохраняете неравенство. Эти свойства указаны в поле ниже.

    Сложение и вычитание свойств неравенства

    Если [латекс] a> b [/ латекс], , то [латекс] a + c> b + c [/ латекс].

    Если [латекс] a> b [/ latex] , , то [латекс] a-c> b-c [/ latex].

    Поскольку неравенство имеет несколько возможных решений, графическое представление решений дает полезную визуализацию ситуации, как мы видели в последнем разделе.В приведенном ниже примере показаны шаги для решения и графического представления неравенства и выражения решения с использованием интервальной записи.

    Пример

    Решите относительно x.

    [латекс] {x} +3 \ lt {5} [/ латекс]

    Показать решение

    Полезно думать об этом неравенстве как о том, что вам предлагается найти все значения для x , включая отрицательные числа, так что при сложении трех вы получите число меньше 5.

    [латекс] \ displaystyle \ begin {array} {l} x + 3 <\, \, \, \, 5 \\\ подчеркивание {\, \, \, \, \, - 3 \, \, \, \, - 3} \\ x \, \, \, \, \, \, \, \, <\, \, \, \, 2 \, \, \ end {array} [/ latex]

    Выделите переменную, вычтя 3 из обеих частей неравенства.

    Ответ

    Неравенство: [латекс] x <2 [/ латекс]

    Интервал: [латекс] \ влево (- \ infty, 2 \ вправо) [/ латекс]

    График:

    Линия представляет всех чисел, к которым можно сложить 3 и получить число меньше 5. Есть много чисел, которые разрешают это неравенство!

    Так же, как вы можете проверить решение уравнения, вы можете проверить решение неравенства. Сначала вы проверяете конечную точку, подставляя ее в соответствующее уравнение.Затем вы проверяете, верно ли неравенство, подставляя любое другое решение, чтобы увидеть, является ли оно одним из решений. Поскольку существует несколько решений, рекомендуется проверить более одного из возможных решений. Это также может помочь вам проверить правильность вашего графика.

    В приведенном ниже примере показано, как вы можете проверить, что [latex] x <2 [/ latex] является решением для [latex] x + 3 <5 [/ latex] .

    Пример

    Убедитесь, что [latex] x <2 [/ latex] является решением для [latex] x + 3 <5 [/ latex].

    Показать решение

    Подставьте конечную точку 2 в соответствующее уравнение [латекс] x + 3 = 5 [/ латекс].

    [латекс] \ begin {array} {r} x + 3 = 5 \\ 2 + 3 = 5 \\ 5 = 5 \ end {array} [/ latex]

    Выберите значение меньше 2, например 0, чтобы проверить неравенство. (Это значение будет на затененной части графика.)

    [латекс] \ displaystyle \ begin {array} {r} x + 3 <5 \\ 0 + 3 <5 \\ 3 <5 \ end {array} [/ latex]

    Проверяет!

    [латекс] x <2 [/ latex] - это решение для [latex] x + 3 <5 [/ latex].

    В следующих примерах показаны проблемы неравенства, которые включают операции с отрицательными числами. Также показан график решения неравенства. Не забудьте проверить решение. Это хорошая привычка!

    Пример

    Решить для x : [латекс] x-10 \ leq-12 [/ латекс]

    Показать решение

    Выделите переменную, добавив 10 к обеим сторонам неравенства.

    [латекс] \ displaystyle \ begin {array} {r} x-10 \ le -12 \\\ underline {\, \, \, + 10 \, \, \, \, \, + 10} \\ x \, \, \, \, \, \, \, \, \, \, \ le \, \, \, — 2 \ end {array} [/ latex]

    Ответ

    Неравенство: [латекс] x \ leq-2 [/ latex]
    Интервал: [латекс] \ left (- \ infty, -2 \ right] [/ latex]
    График: Обратите внимание, что используется замкнутый круг, потому что неравенство «меньше или равно» [латекс] \ left (\ leq \ right) [/ latex]. Синяя стрелка нарисована слева от точки [латекс] -2 [/ латекс], потому что это значения меньше, чем [латекс] -2 [/ латекс].

    Проверьте раствор [латекс] x-10 \ leq -12 [/ latex]

    Показать решение

    Подставьте конечную точку [латекс] -2 [/ латекс] в соответствующее уравнение [латекс] x-10 = -12 [/ латекс]

    [латекс] \ displaystyle \ begin {array} {r} x-10 = -12 \, \, \, \\\ text {Does} \, \, \, — 2-10 = -12? \\ — 12 = -12 \, \, \, \ end {array} [/ latex]

    Выберите значение меньше [латекс] -2 [/ латекс], например [латекс] -5 [/ латекс], чтобы проверить неравенство.(Это значение будет на затененной части графика.)

    [латекс] \ displaystyle \ begin {array} {r} x-10 \ le -12 \, \, \, \\\ text {} \, \ text {Is} \, \, — 5-10 \ le -12? \\ — 15 \ le -12 \, \, \, \\\ text {It} \, \ text {проверяет!} \ End {array} [/ latex]

    [latex] x \ leq -2 [/ latex] — это решение [latex] x-10 \ leq -12 [/ latex]

    Пример

    Решите относительно и . [латекс] a-17> -17 [/ латекс]

    Показать решение

    Выделите переменную, добавив 17 к обеим сторонам неравенства.

    [латекс] \ displaystyle \ begin {array} {r} a-17> -17 \\\ подчеркивание {\, \, \, + 17 \, \, \, \, \, + 17} \\ a \ , \, \, \, \, \, \, \, \, \, \,> \, \, \, \, \, \, 0 \ end {array} [/ latex]

    Ответ

    Неравенство: [латекс] \ displaystyle a \, \,> \, 0 [/ latex]

    Интервал: [latex] \ left (0, \ infty \ right) [/ latex] Обратите внимание, как мы используем скобки слева, чтобы показать, что решение не включает 0.

    График: обратите внимание на пустой кружок, чтобы показать, что решение не включает 0.

    Проверьте раствор [латекс] a-17> -17 [/ латекс]

    Показать решение

    [latex] \ displaystyle a \, \,> \, 0 [/ latex] — правильное решение для [latex] a-17> -17 [/ latex]?

    Подставьте конечную точку 0 в соответствующее уравнение.

    [латекс] \ displaystyle \ begin {array} {r} a-17 = -17 \, \, \, \\\ text {Does} \, \, \, 0-17 = -17? \\ — 17 = -17 \, \, \, \ end {array} [/ latex]

    Выберите значение больше 0, например 20, чтобы проверить неравенство. (Это значение будет на затененной части графика.)

    [латекс] \ displaystyle \ begin {array} {r} a-17> -17 \, \, \, \\\ text {Is} \, \, 20-17> -17? \\ 3> -17 \, \, \, \\\\\ text {Проверяет!} \, \, \, \, \ end {array} [/ latex]

    [latex] \ displaystyle a \,> \, 0 [/ latex] — это решение для [latex] a-17> -17 [/ latex]

    Предыдущие примеры показали вам, как решить одношаговое неравенство с переменной в левой части.В следующем видео представлены примеры того, как разрешить однотипное неравенство.

    Что бы вы сделали, если бы переменная находилась в правой части неравенства? В следующем примере вы увидите, как справиться с этим сценарием.

    Пример

    Решить относительно x : [латекс] 4 \ geq {x} +5 [/ латекс]

    Показать решение

    Выделите переменную, добавив 10 к обеим сторонам неравенства.

    [латекс] \ displaystyle \ begin {array} {r} 4 \ geq {x} +5 \\\ подчеркивание {\, \, \, — 5 \, \, \, \, \, — 5} \\ -1 \, \, \, \, \, \, \, \, \, \, \ ge \, \, \, x \ end {array} [/ latex]

    Перепишите неравенство с переменной слева — это упростит запись интервала и построение графика.

    [латекс] x \ le {-1} [/ латекс]

    Обратите внимание, как острая часть неравенства по-прежнему направлена ​​на переменную, поэтому вместо того, чтобы читать как отрицательное значение больше или равно x, теперь оно читается как x меньше или равно отрицательному.

    Ответ

    Неравенство: [latex] x \ le {-1} [/ latex] Это также можно записать как
    Interval: [latex] \ left (- \ infty, -1 \ right] [/ latex]
    График: Обратите внимание, что замкнутый круг используется, потому что неравенство «меньше или равно».Синяя стрелка нарисована слева от точки [latex] -1 [/ latex], потому что это значения меньше, чем [latex] -1 [/ latex].

    Проверьте решение для [latex] 4 \ geq {x} +5 [/ latex]

    Показать решение

    Подставьте конечную точку [латекс] -1 [/ latex] в соответствующее уравнение [латекс] 4 = x + 5 [/ latex]

    [латекс] \ displaystyle \ begin {array} {r} 4 = x + 5 \, \, \, \\\ text {Does} \, \, \, 4 = -1 + 5? \\ — 1 = -1 \, \, \, \ end {array} [/ latex]

    Выберите значение меньше [латекс] -1 [/ латекс], например [латекс] -5 [/ латекс], чтобы проверить неравенство. (Это значение будет на затененной части графика.)

    [латекс] \ displaystyle \ begin {array} {r} 4 \ geq {-5} +5 \, \, \, \\\ text {} \, \ text {Is} \, \, 4 \ ge 0 ? \\\ text {It} \, \ text {проверяет!} \ end {array} [/ latex]

    [latex] x \ le {-1} [/ latex] — это решение [latex] 4 \ geq {x} +5 [/ latex]

    В следующем видео показаны примеры решения неравенств с переменной справа.

    Решите неравенства умножением и делением

    Решение неравенства с переменной, имеющей коэффициент, отличный от 1, обычно включает умножение или деление.Эти шаги подобны решению одношаговых уравнений, включающих умножение или деление, ЗА ИСКЛЮЧЕНИЕМ знака неравенства. Давайте посмотрим, что происходит с неравенством, когда вы умножаете или делите каждую сторону на одно и то же число.

    Начнем с истинного утверждения:

    [латекс] 10> 5 [/ латекс]

    Давайте попробуем еще раз, начав с того же истинного утверждения:

    [латекс] 10> 5 [/ латекс]

    Затем умножьте обе стороны на одинаковое положительное число:

    [латекс] 10 \ cdot 2> 5 \ cdot 2 [/ латекс]

    На этот раз умножьте обе стороны на одно и то же отрицательное число:

    [латекс] 10 \ cdot-2> 5 \\ \, \, \, \, \, \ cdot -2 \, \ cdot-2 [/ латекс]

    20 больше 10, поэтому истинное неравенство сохраняется:

    [латекс] 20> 10 [/ латекс]

    Погодите! [latex] −20 [/ latex] не на больше, чем на [latex] −10 [/ latex], поэтому у вас неверное утверждение.

    [латекс] -20> -10 [/ латекс]

    При умножении на положительное число оставьте знак неравенства как есть! Вы ​​должны «перевернуть» знак неравенства, чтобы утверждение стало верным:

    [латекс] -20 <-10 [/ латекс]

    Внимание! Когда вы умножаете или делите на отрицательное число, «переверните» знак неравенства. Всякий раз, когда вы умножаете или делите обе стороны неравенства на отрицательное число, знак неравенства должен быть перевернут, чтобы утверждение оставалось верным.Эти правила кратко изложены во вставке ниже.


    Свойства неравенства умножения и деления

    Начать с Умножить на Окончательное неравенство
    [латекс] a> b [/ латекс] [латекс] c [/ латекс] [латекс] ac> bc [/ латекс]
    [латекс] a> b [/ латекс] [латекс] -c [/ латекс] [латекс] ac
    Начать с Разделить на Окончательное неравенство
    [латекс] a> b [/ латекс] [латекс] c [/ латекс] [латекс] \ displaystyle \ frac {a} {c}> \ frac {b} {c} [/ latex]
    [латекс] a> b [/ латекс] [латекс] -c [/ латекс] [латекс] \ displaystyle \ frac {a} {c} <\ frac {b} {c} [/ latex]

    Имейте в виду, что вы меняете знак только при умножении и делении на отрицательное число . Если сложить или вычесть на отрицательное число, неравенство останется прежним.

    Пример

    Решите относительно x. [латекс] 3x> 12 [/ латекс]

    Показать решение Разделите обе части на 3, чтобы изолировать переменную.

    [латекс] \ displaystyle \ begin {array} {r} \ underline {3x}> \ underline {12} \\ 3 \, \, \, \, \, \, \, \, \, \, \, \, 3 \\ x> 4 \, \, \, \ end {array} [/ latex]

    Проверьте свое решение, сначала проверив конечную точку 4, а затем проверив другое решение на предмет неравенства.

    [латекс] \ begin {array} {r} 3 \ cdot4 = 12 \\ 12 = 12 \\ 3 \ cdot10> 12 \\ 30> 12 \\\ text {Проверяет!} \ End {array} [/ латекс]

    Ответ

    Неравенство: [латекс] \ displaystyle x> 4 [/ latex]

    Интервал: [латекс] \ left (4, \ infty \ right) [/ latex]

    График:

    Не было необходимости вносить какие-либо изменения в знак неравенства, потому что обе части неравенства были разделены на положительное значение 3. В следующем примере есть деление на отрицательное число, поэтому есть дополнительный шаг в решении!

    Пример

    Решите для x .[латекс] -2x> 6 [/ латекс]

    Показать решение Разделите каждую сторону неравенства на [latex] −2 [/ latex], чтобы изолировать переменную, и измените направление знака неравенства из-за деления на отрицательное число.

    [латекс] \ displaystyle \ begin {array} {r} \ underline {-2x} <\ underline {\, 6 \,} \\ - 2 \, \, \, \, - 2 \, \\ x < -3 \ end {array} [/ latex]

    Проверьте свое решение, сначала проверив конечную точку [latex] −3 [/ latex], а затем проверив другое решение на предмет неравенства.

    [латекс] \ begin {array} {r} -2 \ left (-3 \ right) = 6 \\ 6 = 6 \ -2 \ left (-6 \ right)> 6 \\ 12> 6 \ end {array} [/ latex]

    Проверяет!

    Ответ

    Неравенство: [латекс] \ displaystyle x <-3 [/ latex]

    Интервал: [латекс] \ left (- \ infty, -3 \ right) [/ latex]

    График:
    Поскольку обе стороны неравенства были разделены отрицательным числом [латекс] -2 [/ latex], символ неравенства был изменен с> на <.

    В следующем видео показаны примеры решения одношаговых неравенств с использованием свойства равенства умножения, где переменная находится слева.

    Подумай об этом

    Прежде чем читать решение следующего примера, подумайте, какие свойства неравенств вам, возможно, потребуется использовать для решения неравенства. Чем этот пример отличается от предыдущего? Напишите свои идеи в поле ниже.

    Решите для x .[латекс] — \ frac {1} {2}> — 12x [/ латекс]

    Показать решение

    Это неравенство имеет переменную в правой части, которая отличается от предыдущих примеров. Начните процесс решения, как и раньше, а в конце вы можете переместить переменную влево, чтобы записать окончательное решение.

    Разделите обе стороны на [латекс] -12 [/ латекс], чтобы изолировать переменную. Поскольку вы делите на отрицательное число, вам нужно изменить направление знака неравенства.

    [латекс] \ displaystyle \ begin {array} {l} — \ frac {1} {2} \ gt {-12x} \\\\\ frac {- \ frac {1} {2}} {- 12} \ gt \ frac {-12x} {- 12} \\\ end {array} [/ latex]

    Для деления дроби на целое число необходимо умножить на обратную величину, а обратная величина [latex] -12 [/ latex] будет [latex] \ frac {1} {- 12} [/ latex]

    [латекс] \ displaystyle \ begin {array} {r} \ left (- \ frac {1} {12} \ right) \ left (- \ frac {1} {2} \ right) \ lt \ frac {- 12x} {- 12} \, \, \\\\ \ frac {1} {24} \ lt \ frac {\ cancel {-12} x} {\ cancel {-12}} \\\\ \ frac { 1} {24} \ lt {x} \, \, \, \, \, \, \, \, \, \, \ end {array} [/ latex]

    Ответ

    Неравенство: [latex] \ frac {1} {24} \ lt {x} [/ latex] Это также можно записать с переменной слева как [latex] x \ gt \ frac {1} {24} [ /латекс].Чтобы записать неравенство с переменной слева, нужно немного подумать, но это поможет вам записать интервал и правильно нарисовать график.

    Интервал: [латекс] \ left (\ frac {1} {24}, \ infty \ right) [/ latex]

    График:

    В следующем видео приведены примеры того, как решить неравенство со свойством умножения равенства, где переменная находится справа.

    Объединение свойств неравенства для решения алгебраических неравенств

    Популярная стратегия решения уравнений с выделением переменной также применима к решению неравенств.Путем сложения, вычитания, умножения и / или деления вы можете переписать неравенство так, чтобы переменная находилась с одной стороны, а все остальное — с другой. Как и в случае одношаговых неравенств, решения многоступенчатых неравенств можно изобразить на числовой прямой.

    Пример

    Решите относительно p . [латекс] 4p + 5 <29 [/ латекс]

    Показать решение

    Начните изолировать переменную, вычтя 5 из обеих частей неравенства.

    [латекс] \ displaystyle \ begin {array} {l} 4p + 5 <\, \, \, 29 \\\ подчеркивание {\, \, \, \, \, \, \, \, \, - 5 \, \, \, \, \, - 5} \\ 4p \, \, \, \, \, \, \, \, \, <\, \, 24 \, \, \ end {array} [ / латекс]

    Разделите обе части неравенства на 4, чтобы выразить переменную с коэффициентом 1.

    [латекс] \ begin {array} {l} \ underline {4p} \, <\, \, \ underline {24} \, \, \\\, 4 \, \, \, \, <\, \ , 4 \\\, \, \, \, \, p <6 \ end {array} [/ latex]

    Ответ

    Неравенство: [латекс] p <6 [/ латекс]

    Интервал: [латекс] \ влево (- \ infty, 6 \ вправо) [/ латекс]

    График

    : Обратите внимание на белый кружок в конечной точке 6, чтобы показать, что решения неравенства не включают 6. Значения, где p меньше 6, находятся вдоль числовой линии слева от 6.

    Проверить решение.

    Показать решение

    Проверьте конечную точку 6 в соответствующем уравнении.

    [латекс] \ displaystyle \ begin {array} {r} 4p + 5 = 29 \, \, \, \\\ text {Does} \, \, \, 4 (6) + 5 = 29? \\ 24 + 5 = 29 \, \, \, \\ 29 = 29 \, \, \, \\\ текст {Да!} \, \, \, \, \, \, \ End {array} [/ latex]

    Попробуйте другое значение, чтобы проверить неравенство. Давайте использовать [latex] p = 0 [/ latex].

    [латекс] \ displaystyle \ begin {array} {r} 4p + 5 <29 \, \, \, \\\ text {Is} \, \, \, 4 (0) +5 <29? \\ 0 +5 <29 \, \, \, \\ 5 <29 \, \, \, \\\ text {Да!} \, \, \, \, \, \ End {array} [/ latex]

    [латекс] p <6 [/ латекс] - это решение [латекс] 4p + 5 <29 [/ латекс]

    Пример

    Решить для x : [латекс] 3x – 7 \ ge 41 [/ латекс]

    Показать решение

    Начните изолировать переменную, прибавив 7 к обеим сторонам неравенства, затем разделите обе стороны неравенства на 3, чтобы выразить переменную с коэффициентом 1.

    [латекс] \ displaystyle \ begin {array} {l} 3x-7 \ ge 41 \\\ подчеркивание {\, \, \, \, \, \, \, + 7 \, \, \, \, + 7} \\\ frac {3x} {3} \, \, \, \, \, \, \, \, \ ge \ frac {48} {3} \\\, \, \, \, \, \, \, \, \, \, x \ ge 16 \ end {array} [/ latex]

    Ответ

    Неравенство: [латекс] x \ ge 16 [/ латекс]

    Интервал: [латекс] \ left [16, \ infty \ right) [/ latex]

    График: Чтобы изобразить это неравенство, вы рисуете замкнутый круг в конечной точке 16 числовой линии, чтобы показать, что решения включают значение 16. Линия продолжается вправо от 16, потому что все числа больше 16 также будут составлять неравенство [латекс] 3x – 7 \ ge 41 [/ латекс] верно.

    Проверить решение.

    Показать решение

    Сначала проверьте конечную точку 16 в соответствующем уравнении.

    [латекс] \ displaystyle \ begin {array} {r} 3x-7 = 41 \, \, \, \\\ text {Does} \, \, \, 3 (16) -7 = 41? \\ 48 -7 = 41 \, \, \, \\ 41 = 41 \, \, \, \\\ текст {Да!} \, \, \, \, \, \ End {array} [/ latex]

    Затем попробуйте другое значение, чтобы проверить неравенство. Давайте использовать [latex] x = 20 [/ latex].

    [латекс] \ displaystyle \ begin {array} {r} \, \, \, \, 3x-7 \ ge 41 \, \, \, \\\ текст {Is} \, \, \, \, \ , 3 (20) -7 \ ge 41? \\ 60-7 \ ge 41 \, \, \, \\ 53 \ ge 41 \, \, \, \\\ текст {Да!} \, \, \ , \, \, \ end {array} [/ latex]

    При решении многоступенчатых уравнений обратите внимание на ситуации, в которых вы умножаете или делите на отрицательное число.В этих случаях необходимо перевернуть знак неравенства.

    Пример

    Решите относительно p . [латекс] −58> 14−6p [/ латекс]

    Показать решение

    Обратите внимание, что переменная находится в правой части неравенства, метод решения в этом случае не меняется.

    Начните изолировать переменную, вычтя 14 из обеих частей неравенства.

    [латекс] \ displaystyle \ begin {array} {l} −58 \, \,> 14−6p \\\ подчеркивание {\, \, \, \, \, \, \, \, \, \, \ , \, — 14 \, \, \, \, \, \, \, — 14} \\ — 72 \, \, \, \, \, \, \, \, \, \, \,> — 6p \ end {array} [/ latex]

    Разделите обе части неравенства на [латекс] −6 [/ латекс], чтобы выразить переменную с коэффициентом 1.При делении на отрицательное число знак неравенства меняется на противоположный.

    [латекс] \ begin {array} {l} \ underline {-72}> \ underline {-6p} \\ — 6 \, \, \, \, \, \, \, \, \, \, — 6 \\\, \, \, \, \, \, 12 \ lt {p} \ end {array} [/ latex]

    Мы также можем записать это как [latex] p> 12 [/ latex]. Обратите внимание, как знак неравенства все еще открывается в сторону переменной p.

    Ответ

    Неравенство: [латекс] p> 12 [/ latex]
    Интервал: [latex] \ left (12, \ infty \ right) [/ latex]
    График: График неравенства p > 12 имеет открытый кружок на 12 со стрелкой, уходящей вправо.

    Проверить решение.

    Показать решение

    Сначала проверьте конечную точку 12 в соответствующем уравнении.

    [латекс] \ begin {array} {r} -58 = 14-6p \\ — 58 = 14-6 \ left (12 \ right) \\ — 58 = 14-72 \\ — 58 = -58 \ end {array} [/ latex]

    Затем попробуйте другое значение, чтобы проверить неравенство. Попробуйте 100.

    [латекс] \ begin {array} {r} -58> 14-6p \\ — 58> 14-6 \ left (100 \ right) \\ — 58> 14-600 \\ — 58> -586 \ end {array} [/ latex]

    В следующем видео вы увидите пример решения линейного неравенства с переменной в левой части неравенства и пример переключения направления неравенства после деления на отрицательное число.

    В следующем видео вы увидите пример решения линейного неравенства с переменной в правой части неравенства и пример переключения направления неравенства после деления на отрицательное число.

    Упростите и решите алгебраические неравенства, используя свойство распределенности

    Как и в случае с уравнениями, свойство распределения может применяться для упрощения выражений, являющихся частью неравенства.Как только скобки будут убраны, устранение неравенства будет несложным.

    Пример

    Решите для x . [латекс] 2 \ влево (3x – 5 \ вправо) \ leq 4x + 6 [/ латекс]

    Показать решение

    Раздать, чтобы убрать скобки.

    [латекс] \ displaystyle \ begin {array} {r} \, 2 (3x-5) \ leq 4x + 6 \\\, \, \, \, 6x-10 \ leq 4x + 6 \ end {array} [/ латекс]

    Вычтите 4 x с обеих сторон, чтобы получить переменный член только с одной стороны.

    [латекс] \ begin {array} {r} 6x-10 \ le 4x + 6 \\\ подчеркивание {-4x \, \, \, \, \, \, \, \, \, \, \, \ , \, \, — 4x} \, \, \, \, \, \, \, \, \, \\\, \, \, 2x-10 \, \, \ leq \, \, \, \ , \, \, \, \, \, \, \, \, 6 \ end {array} [/ latex]

    Добавьте 10 к обеим сторонам, чтобы изолировать переменную.

    [латекс] \ begin {array} {r} \\\, \, \, 2x-10 \, \, \ le \, \, \, \, \, \, \, \, 6 \, \, \, \\\ подчеркивание {\, \, \, \, \, \, + 10 \, \, \, \, \, \, \, \, \, + 10} \\\, \, \, 2x \, \, \, \, \, \, \, \, \, \, \, \ le \, \, \, \, \, 16 \, \, \, \ end {array} [/ latex ]

    Разделите обе части на 2, чтобы выразить переменную с коэффициентом 1.

    [латекс] \ begin {array} {r} \ underline {2x} \ le \, \, \, \ underline {16} \\\, \, \, 2 \, \, \, \, \, \ , \, \, \, \, \, \, \, 2 \, \, \\\, \, \, \, \, \, \, \, \, \, \, \, \, \, \, x \, \, \, \ le \, \, \, \, \, 8 \ end {array} [/ latex]

    Ответ

    Неравенство: [latex] x \ le8 [/ latex]
    Интервал: [latex] \ left (- \ infty, 8 \ right] [/ latex]
    График: График этого набора решений включает 8 и все, что слева от 8 в числовой строке.

    Проверить решение.

    Показать решение

    Сначала проверьте конечную точку 8 в соответствующем уравнении.

    [латекс] \ displaystyle \ begin {array} {r} 2 (3x-5) = 4x + 6 \, \, \, \, \, \, \\ 2 (3 \, \ cdot \, 8-5 ) = 4 \, \ cdot \, 8 + 6 \\\, \, \, \, \, \, \, \, \, \, \, 2 (24-5) = 32 + 6 \, \, \, \, \, \, \\ 2 (19) = 38 \, \, \, \, \, \, \, \, \, \, \, \, \, \, \, \, \\ 38 = 38 \, \, \, \, \, \, \, \, \, \, \, \, \, \, \, \, \ end {array} [/ latex]

    Затем выберите другое решение и оцените неравенство для этого значения, чтобы убедиться, что это истинное утверждение. Попробуйте 0.

    [латекс] \ displaystyle \ begin {array} {l} 2 (3 \, \ cdot \, 0-5) \ le 4 \, \ cdot \, 0 + 6? \\\, \, \, \, \, \, \, \, \, \, \, \, \, \, \, 2 (-5) \ le 6 \\\, \, \, \, \, \, \, \, \, \, \, \, \, \, \, \, \, — 10 \ le 6 \, \, \ end {array} [/ latex]

    [latex] x \ le8 [/ latex] — это решение для [latex] \ left (- \ infty, 8 \ right] [/ latex]

    В следующем видео вам дается пример того, как решить многоступенчатое неравенство, которое требует использования свойства распределения.

    Подумай об этом

    В следующем примере вам дано неравенство с термином, который выглядит сложным.Если вы остановитесь и подумаете о том, как использовать порядок операций для устранения неравенства, мы надеемся, что это покажется простой проблемой. Используйте текстовое поле, чтобы записать, что вы считаете лучшим первым шагом.

    Найдите файл. [латекс] \ displaystyle \ frac {{2} {a} — {4}} {{6}} {<2} [/ latex]

    Показать решение

    Очистите дробь, умножив обе части уравнения на 6.

    [латекс] \ displaystyle \ begin {array} {r} \ frac {{2} {a} — {4}} {{6}} {<2} \, \, \, \, \, \, \ , \, \\\\ 6 \, \ cdot \, \ frac {2a-4} {6} <2 \, \ cdot \, 6 \\\\ {2a-4} <12 \, \, \, \, \, \, \ end {array} [/ latex]

    Добавьте 4 к обеим сторонам, чтобы изолировать переменную.

    [латекс] \ displaystyle \ begin {array} {r} 2a-4 <12 \\\ underline {\, \, \, + 4 \, \, \, \, + 4} \\ 2a <16 \ end {array} [/ latex]

    Разделите обе части на 2, чтобы выразить переменную с коэффициентом 1.

    [латекс] \ displaystyle \ begin {array} {c} \ frac {2a} {2} <\, \ frac {16} {2} \\\\ a <8 \ end {array} [/ latex]

    Ответ

    Неравенство: [латекс] a <8 [/ латекс]

    Интервал: [латекс] \ влево (- \ infty, 8 \ вправо) [/ латекс]

    График: График этого решения содержит сплошную точку у 8, чтобы показать, что 8 входит в набор решений. Линия продолжается влево, чтобы показать, что значения меньше 8 также включены в набор решений.

    Проверить решение.

    Показать решение Сначала проверьте конечную точку 8 в соответствующем уравнении.

    [латекс] \ displaystyle \ begin {array} {r} \ frac {2a-4} {6} = 2 \, \, \, \, \\\\\ text {Does} \, \, \, \ frac {2 (8) -4} {6} = 2? \\\\\ frac {16-4} {6} = 2 \, \, \, \, \\\\\ frac {12} {6 } = 2 \, \, \, \, \\\\ 2 = 2 \, \, \, \, \\\\\ text {Да!} \, \, \, \, \, \ End {массив } [/ латекс]

    Затем выберите другое решение и оцените неравенство для этого значения, чтобы убедиться, что это истинное утверждение.Попробуйте 5.

    [латекс] \ displaystyle \ begin {array} {r} \ text {Is} \, \, \, \ frac {2 (5) -4} {6} <2? \\\\\ frac {10- 4} {6} <2 \, \, \, \\\\\, \, \, \, \ frac {6} {6} <2 \, \, \, \\\\ 1 <2 \, \, \, \\\\\ text {Да!} \, \, \, \, \, \ end {array} [/ latex]

    Сводка

    Решение неравенств очень похоже на решение уравнений, за исключением того, что вам нужно перевернуть символы неравенства, когда вы умножаете или делите обе стороны неравенства на отрицательное число. Решения неравенств можно представить тремя способами: интервал, график и неравенство.Поскольку обычно существует более одного решения неравенства, когда вы проверяете свой ответ, вы должны проверить конечную точку и одно другое значение, чтобы проверить направление неравенства.

    На неравенства могут быть разные ответы. Решения часто отображаются в виде числовой линии, чтобы наглядно представить все решения. Многоступенчатые неравенства решаются с использованием тех же процессов, которые работают для решения уравнений, за одним исключением. Когда вы умножаете или делите обе стороны неравенства на отрицательное число, вы должны перевернуть символ неравенства.Символы неравенства остаются неизменными при добавлении или вычитании положительных или отрицательных чисел к обеим сторонам неравенства.

    Решение квадратичных неравенств

    … и более …

    Квадратичный

    Квадратное уравнение (в стандартной форме) выглядит так:


    Квадратичное уравнение в стандартной форме
    ( a , b и c могут иметь любое значение, за исключением того, что a не может быть 0. )

    Выше приведено уравнение (=), но иногда нам нужно решить такие неравенства:

    Обозначение

    Слова

    Пример

    >

    больше

    x 2 + 3x> 2

    <

    менее

    7x 2 <28

    больше или равно

    5 ≥ х 2 — х

    меньше или равно

    2 года 2 911 10 + 1 ≤ 7 лет

    Решение

    Решение неравенств очень похоже на решение уравнений… мы делаем почти то же самое.

    При решении уравнения мы пытаемся найти точки ,
    , например, помеченные «= 0»
    Но когда мы решаем неравенства мы пытаемся найти интервала (ов) ,
    , например, помеченные «> 0» или «<0"

    Итак, что мы делаем:

    • найти «= 0» точки
    • между точками «= 0» — это интервала, либо
      • больше нуля (> 0) или
      • меньше нуля (<0)
    • , затем выберите тестовое значение, чтобы узнать, какое оно (> 0 или <0)

    Вот пример:

    Пример: x

    2 — x — 6 <0

    x 2 — x — 6 имеет следующие простые коэффициенты (потому что я хотел упростить!):

    (х + 2) (х − 3) <0

    Для начала , найдем где равно ноль:

    (х + 2) (х − 3) = 0

    Равно нулю, когда x = −2 или x = +3.
    , потому что, когда x = −2, тогда (x + 2) равно нулю
    или когда x = +3, то (x − 3) равно нулю

    Итак, между −2 и +3, функция будет либо

    • всегда больше нуля или
    • всегда меньше чем ноль

    Мы не знаем какой… еще!

    Давайте выберем промежуточное значение и проверим его:

    При x = 0: x 2 — x — 6

    = 0–0–6

    = −6

    Таким образом, между -2 и +3 функция будет на меньше, чем на .

    И это именно тот регион, который нам нужен, так что …

    x 2 — x — 6 <0 в интервале (−2, 3)

    Примечание: x 2 — x — 6> 0 на интервале (−∞, −2) и (3, + ∞)

    А вот график x 2 — x — 6:

    • Уравнение равно нулю при −2 и 3
    • Неравенство «<0» верно между −2 и 3.

    Также попробуйте Grapher неравенства.

    Что делать, если он не проходит через ноль?

    Вот график x 2 — x + 1

    Нет баллов «= 0»!

    Но от этого все становится проще!

    Поскольку линия не пересекает y = 0, она должна быть либо:

    • всегда> 0 или
    • всегда <0

    Итак, все, что нам нужно сделать, это проверить одно значение (скажем, x = 0), чтобы увидеть, выше или ниже оно.

    A Пример «Реальный мир»

    Каскадер прыгнет с 20-метрового здания.

    Высокоскоростная камера готова снимать его на высоте от 15 до 10 метров над землей.

    Когда камера должна его снимать?

    Мы можем использовать эту формулу для расстояния и времени:

    d = 20-5 т 2

    • d = расстояние от земли (м) и
    • t = время от прыжка (секунды)

    (Примечание: если вам интересна формула, она упрощена из d = d 0 + v 0 t + ½a 0 t 2 , где d 0 = 20 , v 0 = 0 и a 0 = −9.81 , г. в ускорение свободного падения.)

    Хорошо, поехали.

    Сначала , давайте набросаем вопрос:

    Требуемое расстояние от 10 м до 15 м :

    10

    И мы знаем формулу для d:

    10 <20-5 т 2 <15

    Теперь решим!

    Сначала вычтем 20 с обеих сторон:

    −10 <−5t 2 <−5

    Теперь умножьте обе стороны на — (1/5). Но из-за того, что мы умножаем на отрицательное число, неравенства изменят направление… прочтите «Решение неравенств», чтобы понять, почему.

    2> т 2 > 1

    Для наглядности меньшее число должно быть слева, а большее — справа. Так что давайте поменяем их местами (и убедимся, что неравенства по-прежнему указывают правильно):

    1 2 <2

    Наконец, мы можем безопасно извлекать квадратные корни, поскольку все значения больше нуля:

    √1

    Съемочную группу можем сказать:

    «Фильм из 1.От 0 до 1,4 секунды после прыжка «

    Выше квадратичного

    Те же идеи могут помочь нам решить более сложные неравенства:

    Пример: x

    3 + 4 ≥ 3x 2 + x

    Во-первых, приведем его в стандартном виде:

    x 3 — 3x 2 — x + 4 ≥ 0

    Это кубическое уравнение (наивысший показатель — куб, т. Е. X 3 ), и его сложно решить, поэтому давайте изобразим его вместо этого:

    Нулевые точки: приблизительно :

    А на графике мы видим интервалы, где он больше (или равен) нулю:

    • От −1.1 до 1,3 и
    • с 2,9 по

    В интервальных обозначениях можно написать:

    Примерно: [-1,1, 1,3] U [2,9, + ∞)

    Введение в неравенства и интервальную нотацию

    2.7 Введение в неравенства и интервальные обозначения

    Цели обучения

    1. Изобразите решения одного неравенства на числовой прямой и выразите решения в интервальной нотации.
    2. Изобразите решения сложного неравенства на числовой прямой и выразите решения, используя интервальную нотацию.

    Неограниченные интервалы

    Алгебраическое неравенство Выражения, связанные с символами ≤, <, ≥ и>., Например x≥2, читается как « x больше или равно 2». Это неравенство имеет бесконечно много решений для x . Некоторые из решений: 2, 3, 3.5, 5, 20 и 20.001.Поскольку невозможно перечислить все решения, необходима система, позволяющая четко передавать этот бесконечный набор. Два распространенных способа выражения решений неравенства — это их графическое представление на числовой прямой. Решения алгебраического неравенства, выраженные затенением решения на числовой прямой. и с использованием интервальной нотации Текстовая система выражения решений алгебраического неравенства ..

    Чтобы выразить решение графически, нарисуйте числовую линию и заштрихуйте все значения, которые являются решениями неравенства.Обозначение интервалов является текстовым и использует следующие специальные обозначения:

    Определите обозначение интервала после построения графика решения, установленного на числовой прямой. Числа в обозначении интервалов следует записывать в том же порядке, в каком они появляются в числовой строке, при этом меньшие числа в наборе появляются первыми. В этом примере есть инклюзивное неравенство Неравенство, которое включает граничную точку, обозначенную «или равной» частью символов ≤ и ≥, и замкнутую точку на числовой прямой., что означает, что нижняя граница 2 входит в решение. Обозначьте это закрытой точкой на числовой прямой и квадратной скобкой в ​​обозначении интервалов. Символ (∞) читается как бесконечность. Символ (∞) указывает, что интервал неограничен вправо. и указывает, что набор неограничен справа на числовой прямой. Для обозначения интервалов необходимо, чтобы бесконечность заключалась в круглые скобки. Квадратная скобка указывает, что граница включена в решение. Скобки указывают, что граница не включена. Бесконечность — это верхняя граница действительных чисел, но сама по себе не является действительным числом: его нельзя включить в набор решений.

    Теперь сравните обозначение интервала в предыдущем примере с обозначением строгого или неисключительного неравенства, которое следует ниже:

    Строгое неравенство: Выразите отношения упорядочения с помощью символа <для «меньше чем» и> для «больше чем». подразумевают, что решения могут очень близко подходить к граничной точке, в данном случае 2, но фактически не включать ее.Обозначьте эту идею открытой точкой на числовой прямой и круглой скобкой в ​​обозначении интервалов.

    Пример 1: Изобразите график и дайте эквивалент записи интервала: x <3.

    Решение: Используйте точку 3 и закрасьте все действительные числа строго меньше 3. Используйте отрицательную бесконечность. Символ (-∞) указывает, что интервал не ограничен слева. (−∞), чтобы указать, что множество решений неограничено слева на числовой прямой.

    Ответ: Обозначение интервала: (−∞, 3)

    Пример 2: График и дайте интервал эквивалентного обозначения: x≤5.

    Решение: Используйте закрытую точку и заштрихуйте все числа меньше 5 включительно.

    Ответ: Обозначение интервала: (−∞, 5]

    Важно видеть, что 5≥x то же самое, что x≤5. Оба требуют, чтобы значения x были меньше или равны 5.Чтобы избежать путаницы, рекомендуется переписать все неравенства с переменной слева. Кроме того, при использовании текста используйте «inf» как сокращенную форму бесконечности. Например, (−∞, 5] можно текстуально выразить как (−inf, 5].

    Сложное неравенство Два неравенства в одном утверждении, соединенные словом «и» или словом «или». фактически представляет собой два или более неравенства в одном утверждении, соединенных словом «и» или словом «или». Сложные неравенства с логическим «или» требуют выполнения любого из условий.Следовательно, множество решений этого типа сложного неравенства состоит из всех элементов множеств решений каждого неравенства. Когда мы объединяем эти индивидуальные наборы решений, это называется объединением. Множество образовано путем объединения индивидуальных наборов решений, обозначенных логическим использованием слова «или» и обозначенных символом ∪., Обозначенным ∪. Например, решения составного неравенства x <3 или x≥6 можно изобразить следующим образом:

    Иногда мы сталкиваемся с составными неравенствами, когда отдельные наборы решений перекрываются.В случае, когда составное неравенство содержит слово «или», мы объединяем все элементы обоих наборов, чтобы создать один набор, содержащий все элементы каждого из них.

    Пример 3: График и эквивалентное обозначение интервала: x≤ − 1 или x <3.

    Решение: Объедините все решения обоих неравенств. Решения каждого неравенства показаны над числовой линией как средство определения объединения, которое изображено на числовой прямой ниже.

    Ответ: Обозначение интервала: (−∞, 3)

    Любое действительное число меньше 3 в заштрихованной области числовой прямой удовлетворяет по крайней мере одному из двух указанных неравенств.

    Пример 4: Изобразите график и укажите эквивалентную интервальную нотацию: x <3 или x≥ − 1.

    Решение: Оба набора решений изображены над объединением, которое показано ниже.

    Ответ: Обозначение интервала: R = (−∞, ∞)

    Когда вы объединяете оба набора решений и формируете объединение, вы можете видеть, что все действительные числа удовлетворяют исходному составному неравенству.

    Таким образом,

    и

    Ограниченные интервалы

    Неравенство, такое как

    читается как «-1 меньше или равно x , а x меньше трех.”Это сложное неравенство, потому что его можно разложить следующим образом:

    Логическое «и» требует, чтобы выполнялись оба условия. Оба неравенства удовлетворяются всеми элементами в пересечении Множество, образованное общими значениями отдельных наборов решений, что обозначено логическим использованием слова «и», обозначенного символом., Обозначенным, множеств решений. каждого.

    Пример 5: Изобразите график и дайте эквивалент записи интервала: x <3 и x≥ − 1.

    Решение: Определите пересечение или перекрытие двух наборов решений. Решения каждого неравенства показаны над числовой линией как средство определения пересечения, которое показано на числовой прямой ниже.

    Здесь x = 3 не является решением, поскольку решает только одно из неравенств.

    Ответ: Обозначение интервала: [−1, 3)

    В качестве альтернативы мы можем интерпретировать −1≤x <3 как все возможные значения для x между или ограниченные −1 и 3 на числовой прямой.Например, одно из таких решений — x = 1. Обратите внимание, что 1 находится между -1 и 3 на числовой прямой или что -1 <1 <3. Точно так же мы видим, что другие возможные решения - -1, -0,99, 0, 0,0056, 1,8 и 2,99. Поскольку существует бесконечно много действительных чисел между -1 и 3, мы должны выразить решение графически и / или в интервальной записи, в данном случае [-1, 3).

    Пример 6: Изобразите график и укажите эквивалент записи интервала: −32

    Решение: Закрасьте все действительные числа, ограниченные или строго между −32 = −112 и 2.

    Ответ: Обозначение интервала: (−32, 2)

    Пример 7: Изобразите график и дайте эквивалент записи интервала: −5

    Решение: Закрасьте все действительные числа от -5 до 15 и укажите, что верхняя граница, 15, включена в набор решений, с помощью закрытой точки.

    Ответ: Обозначение интервала: (−5, 15]

    В предыдущих двух примерах мы не разбирали неравенства; вместо этого мы решили думать обо всех действительных числах между двумя заданными границами.

    Таким образом,

    Обозначение конструктора множеств

    В этом тексте мы используем обозначение интервалов. Однако другие ресурсы, с которыми вы, вероятно, столкнетесь, используют альтернативный метод описания множеств, называемый нотацией построителя множеств — системой для описания множеств с использованием знакомой математической нотации.. Мы использовали обозначение набора для перечисления элементов, таких как целые числа

    Фигурные скобки группируют элементы набора, а знаки многоточия указывают на то, что целые числа продолжаются бесконечно. В этом разделе мы хотим описать интервалы действительных чисел, например действительных чисел, больших или равных 2.

    Поскольку набор слишком велик для перечисления, нотация конструктора множеств позволяет нам описывать его, используя знакомые математические обозначения.Пример обозначения конструктора множеств:

    Здесь x R описывает тип числа, где символ (∈) читается как «элемент». Это означает, что переменная x представляет собой действительное число. Вертикальная черта (|) читается как «такая, что». Наконец, утверждение x≥2 — это условие, описывающее множество с использованием математической записи. На этом этапе нашего изучения алгебры предполагается, что все переменные представляют действительные числа.По этой причине вы можете опустить «∈ R » и написать {x | x≥2}, что читается как «набор всех действительных чисел x , таких, что x больше или равно 2. ”

    Чтобы описать составные неравенства, такие как x <3 или x≥6, напишите {x | x <3 или x≥6}, что читается как «набор всех действительных чисел x , таких, что x меньше 3. или x больше или равно 6. »

    Запишите ограниченные интервалы, такие как −1≤x <3, как {x | −1≤x <3}, что читается как «набор всех действительных чисел x таких, что x больше или равно −1 и меньше 3.”

    Ключевые выводы

    • Неравенства обычно имеют бесконечно много решений, поэтому вместо того, чтобы представлять невероятно большой список, мы представляем такие наборы решений либо графически на числовой строке, либо текстуально с использованием интервальной нотации.
    • Включающие неравенства с компонентом «или равно» обозначаются закрытой точкой на числовой прямой и квадратной скобкой с использованием обозначения интервала.
    • Строгие неравенства без компонента «или равно» обозначаются открытой точкой в ​​числовой строке и круглыми скобками с использованием обозначения интервала.
    • Сложные неравенства, в которых используется логическое «или», разрешаются решениями любого неравенства. Набор решений — это объединение каждого отдельного набора решений.
    • Сложные неравенства, использующие логическое «и», требуют, чтобы все неравенства решались одним решением.Набор решений — это пересечение каждого отдельного набора решений.
    • Сложные неравенства вида n A ограничен между значениями n и m .

    Тематические упражнения

    Часть A: Простые неравенства

    Изобразите все решения на числовой прямой и укажите соответствующие интервалы.

    1. x≤10

    2. х> −5

    3. x> 0

    4. x≤0

    5. х≤ − 3

    6. x≥ − 1

    7. −4 <х

    8. 1≥x

    9. х <−12

    10. x≥ − 32

    11. x≥ − 134

    12. x <34

    Часть B: Сложные неравенства

    Изобразите все решения на числовой прямой и укажите соответствующие интервалы.

    13. -2 <х <5

    14. −5≤x≤ − 1

    15. −5

    16. 0≤x <15

    17. 10

    18. −40≤x <−10

    19. 0

    20. −30

    21. -58 <х <18

    22. −34≤x≤12

    23. −1≤x <112

    24. -112 <х <-12

    25.x <−3 или x> 3

    26. x <−2 или x≥4

    27. x≤0 или x> 10

    28. x≤ − 20 или x≥ − 10

    29. x <−23 или x> 13

    30. x≤ − 43 или x> −13

    31. x> −5 или x <5

    32. x <12 или x> −6

    33. x <3 или x≥3

    34. x≤0 или x> 0

    35. x <−7 или x <2

    36.x≥ − 3 или x> 0

    37. x≥5 или x> 0

    38. x <15 или x≤10

    39. x> −2 и x <3

    40. x≥0 и x <5

    41. x≥ − 5 и x≤ − 1

    42. x <−4 и x> 2

    43. x≤3 и x> 3

    44. x≤5 и x≥5

    45. x≤0 и x≥0

    46. x <2 и x≤ − 1

    47.х> 0 и х≥ − 1

    48. x <5 и x <2

    Часть C: Обозначение интервалов

    Определите неравенство по ответам, выраженным в виде интервалов.

    49. (-∞, 7]

    50. (-4, ∞)

    51. [-12, ∞)

    52. (−∞, −3)

    53. (-8, 10]

    54. (-20, 0]

    55.(−14, −2)

    56. [23, 43]

    57. (−34, 12)

    58. (−∞, −8)

    59. (8, ∞)

    60. (−∞, 4) ∪ [8, ∞)

    61. (−∞, −2] ∪ [0, ∞)

    62. (−∞, −5] ∪ (5, ∞)

    63. (−∞, 0) ∪ (2, ∞)

    64. (−∞, −15) ∪ (−5, ∞)

    Запишите эквивалентное неравенство.

    65. Все действительные числа меньше 27.

    66. Все действительные числа меньше или равны нулю.

    67. Все действительные числа больше 5.

    68. Все действительные числа больше или равные −8.

    69. Все действительные числа строго между −6 и 6.

    70. Все действительные числа строго между -80 и 0.

    Часть D. Темы дискуссионной доски

    71. Сравните обозначение интервалов с обозначением создателя множеств.Приведите пример набора, описанного с использованием обеих систем.

    72. Объясните, почему мы не используем скобки в обозначении интервалов, когда бесконечность является конечной точкой.

    73. Изучите и обсудите различные сложные формы неравенства, особенно союзы и пересечения.

    74. Изучите и обсудите историю бесконечности.

    75. Изучите и обсудите вклад Георга Кантора.

    76. Что такое диаграмма Венна? Объясните и опубликуйте пример.

    ответы

    1: (−∞, 10]

    3: (0, ∞)

    5: (−∞, −3]

    7: (−4, ∞)

    9: (-∞, -12)

    11: [-134, ∞)

    13: (-2, 5)

    15: (-5, 20]

    17: (10, 40]

    19: (0, 50]

    21: (-58, 18)

    23: [-1, 112)

    25: (−∞, −3) ∪ (3, ∞)

    27: (−∞, 0] ∪ (10, ∞)

    29: (−∞, −23) ∪ (13, ∞)

    31: рэнд

    33: R

    35: (−∞, 2)

    37: (0, ∞)

    39: (-2; 3)

    41: [−5, −1]

    43: ∅

    45: {0}

    47: (0, ∞)

    49: х≤7

    51: x≥ − 12

    53: −8

    55: −14

    57: -34 <х <12

    59: x> 8

    61: x≤ − 2 или x≥0

    63: x <0 или x> 2

    65: x <27

    67: x> 5

    69: −6

    Решение полиномиальных неравенств

    Решение полиномиальных неравенств Вот шаги, необходимые для решения полиномиальных неравенств:
    Шаг 1 : Напишите многочлен в правильной форме.Многочлен должен быть записан в порядке убывания и должен быть меньше, больше, меньше или равен или больше или равен нулю.
    Шаг 2 : Найдите ключевые или критические значения. Чтобы найти ключевые / критические значения, приравняйте уравнение к нулю и решите.
    Шаг 3 : Сделайте диаграмму анализа знаков. Чтобы создать диаграмму анализа знаков, используйте ключевые / критические значения, найденные на шаге 2, чтобы разделить числовую линию на участки.
    Шаг 4 : Выполните анализ знаков. Чтобы провести анализ знаков, выберите по одному числу из каждого раздела, созданного на шаге 3, и вставьте это число в многочлен, чтобы определить знак полученного ответа. Знак этого ответа (положительный или отрицательный) будет знаком всего раздела. Вы можете проверить другой номер из того же раздела, если хотите проверить свой ответ.
    Шаг 5 : Используйте диаграмму знакового анализа, чтобы определить, какие разделы удовлетворяют неравенству.Если неравенство меньше нуля или меньше или равно нулю, тогда вы хотите, чтобы все отрицательные участки были найдены в диаграмме анализа знаков. Если неравенство больше нуля или больше или равно нулю, тогда вы хотите, чтобы все положительные участки были найдены в диаграмме анализа знаков.
    Шаг 6 : Используйте интервальную запись, чтобы написать окончательный ответ.

    Пример 1 — График:

    Шаг 1 : Запишите многочлен в правильной форме.Многочлен должен быть записан в порядке убывания и должен быть меньше, больше, меньше или равен или больше или равен нулю.
    Шаг 2 : Найдите ключевые или критические значения. Чтобы найти ключевые / критические значения, приравняйте уравнение к нулю и решите.
    Шаг 3 : Сделайте диаграмму анализа знаков. Чтобы создать диаграмму анализа знаков, используйте ключевые / критические значения, найденные на шаге 2, чтобы разделить числовую линию на участки.
    Шаг 4 : Выполните анализ знаков. Чтобы провести анализ знаков, выберите по одному числу из каждого раздела, созданного на шаге 3, и вставьте это число в многочлен, чтобы определить знак полученного ответа. В этом случае вы можете выбрать x = –3, что дает +7, x = 0, что дает –8, и x = 5, что дает +7.
    Шаг 5 : Используйте диаграмму анализа знаков, чтобы определить, какие разделы удовлетворяют неравенству.В этом случае у нас больше или равно нулю, поэтому нам нужны все положительные секции.
    Шаг 6 : Используйте интервальную нотацию для записи окончательного ответа.

    Пример 2 — График:

    Шаг 1 : Запишите многочлен в правильной форме. Многочлен должен быть записан в порядке убывания и должен быть меньше, больше, меньше или равен или больше или равен нулю.
    Шаг 2 : Найдите ключевые или критические значения. Чтобы найти ключевые / критические значения, приравняйте уравнение к нулю и решите.
    Шаг 3 : Сделайте диаграмму анализа знаков. Чтобы создать диаграмму анализа знаков, используйте ключевые / критические значения, найденные на шаге 2, чтобы разделить числовую линию на участки.
    Шаг 4 : Выполните анализ знаков.Чтобы провести анализ знаков, выберите по одному числу из каждого раздела, созданного на шаге 3, и вставьте это число в многочлен, чтобы определить знак полученного ответа. В этом случае вы можете выбрать x = –4, что дает +3, x = –2, что дает –1, и x = 0, что дает +3.
    Шаг 5 : Используйте диаграмму анализа знаков, чтобы определить, какие разделы удовлетворяют неравенству. В этом случае у нас меньше или равно нулю, поэтому нам нужна отрицательная секция.
    Шаг 6 : Используйте интервальную нотацию для записи окончательного ответа.

    Нажмите здесь, чтобы узнать о проблеме

    Пример 3 — График:

    Шаг 1 : Запишите многочлен в правильной форме. Многочлен должен быть записан в порядке убывания и должен быть меньше, больше, меньше или равен или больше или равен нулю.
    Шаг 2 : Найдите ключевые или критические значения. Чтобы найти ключевые / критические значения, приравняйте уравнение к нулю и решите.
    Шаг 3 : Сделайте диаграмму анализа знаков. Чтобы создать диаграмму анализа знаков, используйте ключевые / критические значения, найденные на шаге 2, чтобы разделить числовую линию на участки.
    Шаг 4 : Выполните анализ знаков.Чтобы провести анализ знаков, выберите по одному числу из каждого раздела, созданного на шаге 3, и вставьте это число в многочлен, чтобы определить знак полученного ответа. В этом случае вы можете выбрать x = –4, что дает –35, x = 0, что дает +9, x = 2, что дает –5, и x = 4, что дает +21.
    Шаг 5 : Используйте диаграмму анализа знаков, чтобы определить, какие разделы удовлетворяют неравенству. В этом случае у нас меньше нуля, поэтому нам нужны все отрицательные секции.
    Шаг 6 : Используйте интервальную нотацию для записи окончательного ответа.

    Нажмите здесь, чтобы узнать о проблеме

    Пример 4 — График:

    Шаг 1 : Запишите многочлен в правильной форме. Многочлен должен быть записан в порядке убывания и должен быть меньше, больше, меньше или равен или больше или равен нулю.
    Шаг 2 : Найдите ключевые или критические значения. Чтобы найти ключевые / критические значения, приравняйте уравнение к нулю и решите.
    Шаг 3 : Сделайте диаграмму анализа знаков. Чтобы создать диаграмму анализа знаков, используйте ключевые / критические значения, найденные на шаге 2, чтобы разделить числовую линию на участки.
    Шаг 4 : Выполните анализ знаков.Чтобы провести анализ знаков, выберите по одному числу из каждого раздела, созданного на шаге 3, и вставьте это число в многочлен, чтобы определить знак полученного ответа. В этом случае вы можете выбрать x = –5, что дает +56, x = –2, что дает –16, x = 0, что дает +16, и x = 3, что дает –56.
    Шаг 5 : Используйте диаграмму анализа знаков, чтобы определить, какие разделы удовлетворяют неравенству. В этом случае у нас больше нуля, поэтому нам нужны все положительные секции.
    Шаг 6 : Используйте интервальную нотацию для записи окончательного ответа.

    Нажмите здесь, чтобы узнать о проблеме

    2.5 Решение линейных неравенств — промежуточная алгебра 2e

    Цели обучения

    К концу этого раздела вы сможете:

    • График неравенств на числовой прямой
    • Решите линейные неравенства
    • Переведите слова в неравенство и решите
    • Решите приложения с линейными неравенствами

    Будьте готовы 2.13

    Прежде чем вы начнете, пройдите тест на готовность.

    Перевести с алгебры на английский: 15> x.15> x.
    Если вы пропустили эту проблему, просмотрите Пример 1.3.

    Будьте готовы 2.14

    Переведите в алгебраическое выражение: 15 меньше x .
    Если вы пропустили эту проблему, просмотрите Пример 1.8.

    Неравенства в графике на числовой прямой

    Какое число сделало бы неравенство x> 3x> 3 истинным? Вы думаете: « x может быть четыре»? Это правильно, но x тоже может быть 6, 37 или даже 3.001. Любое число больше трех является решением неравенства x> 3.x> 3.

    Мы показываем все решения неравенства x> 3x> 3 на числовой прямой, закрашивая все числа справа от трех, чтобы показать, что все числа больше трех являются решениями. Поскольку число три само по себе не является решением, мы заключили тройку в открывающую скобку.

    Мы также можем представить неравенства, используя обозначение интервала . У решения этого неравенства нет верхнего предела.В обозначениях интервалов мы выразим x> 3x> 3 как (3, ∞). (3, ∞). Символ ∞∞ читается как « бесконечность ». Это не настоящее число.

    На рис. 2.2 показаны числовая линия и интервал.

    Рис. 2.2 Неравенство x> 3x> 3 изображено на этой числовой прямой и записано в интервальных обозначениях.

    Мы используем символ левой круглой скобки (, чтобы показать, что конечная точка неравенства не включена. Символ левой скобки, [, показывает, что конечная точка включена.

    Неравенство x≤1x≤1 означает, что все числа меньше или равны единице. Здесь нам нужно показать, что это тоже решение. Мы делаем это, помещая скобку в x = 1.x = 1. Затем мы закрашиваем все числа слева от единицы, чтобы показать, что все числа меньше единицы являются решениями. См. Рисунок 2.3.

    У этих чисел нет нижнего предела. Мы пишем x≤1x≤1 в обозначении интервалов как (−∞, 1]. (- ∞, 1]. Символ −∞ − ∞ читается как «отрицательная бесконечность». На рисунке 2.3 показаны как числовая строка, так и обозначение интервала.

    Рис. 2.3 На этой числовой прямой изображено неравенство x≤1x≤1 и записано в интервальной записи.

    Неравенства, числовые линии и обозначение интервалов

    В обозначении неравенств на числовой строке и в обозначении интервалов используются одни и те же символы для обозначения конечных точек интервалов.

    Пример 2.48

    Изобразите каждое неравенство в числовой строке и запишите в интервале.

    ⓐ x≥ − 3x≥ − 3 ⓑ x <2,5x <2,5 ⓒ x≤ − 35x≤ − 35

    Попробуй 2.95

    Изобразите каждое неравенство на числовой прямой и запишите в интервальной записи: ⓐ x> 2x> 2 ⓑ x≤ − 1,5x≤ − 1,5 ⓒ x≥34.x≥34.

    Попробуйте 2.96

    Изобразите каждое неравенство на числовой прямой и запишите в интервальной записи: ⓐ x≤ − 4x≤ − 4 ⓑ x≥0,5x≥0,5 ⓒ x <−23.x <−23.

    Какие числа больше двух, но меньше пяти? Вы думаете, скажем, 2,5,3,323,4,4,99? 2,5,3,323,4,4,99? Мы можем представить все числа от двух до пяти неравенством 2

    Рисунок 2.4

    Пример 2.49

    Изобразите каждое неравенство в числовой строке и запишите в интервале.

    ⓐ −3

    Попробуйте 2.97

    Постройте каждое неравенство в числовой строке и запишите в интервале обозначений:

    ⓐ −2

    Попробовать 2,98

    Постройте каждое неравенство в числовой строке и запишите в интервале обозначений:

    ⓐ −6

    Решите линейные неравенства

    Линейное неравенство во многом похоже на линейное уравнение, но знак равенства заменен знаком неравенства. Линейное неравенство — это неравенство с одной переменной, которое может быть записано в одной из форм: ax + b c , ax + b> c или ax + b≥c.топор + b≥c.

    Линейное неравенство

    Линейное неравенство — это неравенство в одной переменной, которое может быть записано в одной из следующих форм, где a , b и c — действительные числа и a 0a ≠ 0:

    ax + b c, ax + b≥c.ax + b c, ax + b≥c.

    Когда мы решали линейные уравнения, мы могли использовать свойства равенства, чтобы складывать, вычитать, умножать или делить обе части и при этом сохранять равенство. Аналогичные свойства верны и для неравенств.

    Мы можем прибавить или вычесть одну и ту же величину из обеих частей неравенства и при этом сохранить неравенство. Например:

    Обратите внимание, что знак неравенства остался прежним.

    Это приводит нас к свойствам сложения и вычитания неравенства.

    Свойство неравенства сложения и вычитания

    Для любых чисел a , b и c, , если a a + c Для любых чисел a , b и c, если a> b, то a> b, то

    a + c> b + ca − c> b − ca + c> b + ca − c> b − c

    Мы можем прибавить или вычесть одну и ту же величину из обеих частей неравенства и при этом сохранить неравенство.

    Что происходит с неравенством, когда мы делим или умножаем обе части на константу?

    Давайте сначала умножим и разделим обе части на положительное число.

    Признаки неравенства остались прежними.

    Неравенство сохраняется, когда мы делим или умножаем на отрицательное число?

    Обратите внимание, что когда мы заполняли знаки неравенства, знаки неравенства меняли свое направление.

    Когда мы делим или умножаем неравенство на положительное число, знак неравенства остается прежним.Когда мы делим или умножаем неравенство на отрицательное число, знак неравенства меняется на противоположный.

    Это дает нам свойство неравенства умножения и деления.

    Свойство неравенства умножения и деления

    Для любых номеров a , b и c ,

    умножить или разделить на положительное значение, если a 0, затем ac bandc> 0, затем ac> bcandac> bc. умножить или разделить на отрицательное значение ifa bcandac> bc.ifa> bandc <0, thenac 0, thenac bandc> 0, thenac> bcandac> bc. умножить или разделить на отрицательныйifa bcandac> bc.ifa> bandc <0, тогда ac Когда мы делим или умножаем неравенство на , получаем :

    • положительное число, неравенство остается прежним.
    • отрицательное число, неравенство обратное.

    Иногда при решении неравенства, как в следующем примере, переменная заканчивается справа.Мы можем переписать неравенство в обратном порядке, чтобы переменная оказалась слева.

    x> a имеет то же значение, что и a a имеет то же значение, что и a Думайте об этом так: «Если Ксандер выше Энди, то Энди ниже Ксандера».

    Пример 2,50

    Решите каждое неравенство. Постройте решение на числовой прямой и запишите решение в виде интервалов.

    ⓐ x − 38≤34x − 38≤34 ⓑ 9y <549y <54 ⓒ −15 <35z − 15 <35z

    Попробовать 2,99

    Решите каждое неравенство, нанесите решение на числовой прямой и запишите решение в интервальной записи:

    ⓐ p − 34≥16p − 34≥16 ⓑ 9c> 729c> 72 ⓒ 24≤38m24≤38m

    Попробуй 2.100

    Решите каждое неравенство, нанесите решение на числовой прямой и запишите решение в интервальной записи:

    ⓐ r − 13≤712r − 13≤712 ⓑ 12d≤ 6012d≤ 60 ⓒ −24 <43n − 24 <43n

    Будьте осторожны при умножении или делении на отрицательное число — не забудьте перевернуть знак неравенства.

    Пример 2.51

    Решите каждое неравенство, нанесите решение на числовой прямой и запишите решение в интервальной записи.

    ⓐ −13m≥65−13m≥65 ⓑ n − 2≥8n − 2≥8

    Попробуй 2.101

    Решите каждое неравенство, нанесите решение на числовой прямой и запишите решение в интервальной записи:

    ⓐ −8q <32−8q <32 ⓑ k − 12≤15.k − 12≤15.

    Попробуйте 2.102

    Решите каждое неравенство, нанесите решение на числовой прямой и запишите решение в интервальной записи:

    ⓐ −7r≤ −70−7r≤ −70 ⓑ u − 4≥ − 16.u − 4≥ − 16.

    Для устранения большинства неравенств потребуется более одного шага. Мы следуем тем же шагам, что и в общей стратегии решения линейных уравнений, но обязательно обращаем особое внимание при умножении или делении, чтобы изолировать переменную.

    Пример 2.52

    Решите неравенство 6y≤11y + 17,6y≤11y + 17, обозначьте решение на числовой прямой и запишите решение в интервальной записи.

    Попробуйте 2.103

    Решите неравенство, отобразите решение на числовой прямой и запишите решение в интервальной записи: 3q≥7q − 23.3q≥7q − 23.

    Попробуйте 2.104

    Решите неравенство, отобразите решение на числовой прямой и запишите решение в интервальной записи: 6x <10x + 19.6x <10x + 19.

    При решении неравенств обычно проще всего собрать переменные на той стороне, где коэффициент переменной наибольший. Это исключает отрицательные коэффициенты, и нам не нужно умножать или делить на отрицательные значения, а это означает, что нам не нужно помнить о том, чтобы поменять местами знак неравенства.

    Пример 2.53

    Решите неравенство 8p + 3 (p − 12)> 7p − 28,8p + 3 (p − 12)> 7p − 28, обозначьте решение на числовой прямой и запишите решение в интервальной записи.

    Решение
    8p + 3 (p − 12)> 7p − 288p + 3 (p − 12)> 7p − 28
    Максимально упростите каждую сторону.
    Распространить. 8p + 3p − 36> 7p − 288p + 3p − 36> 7p − 28
    Объедините похожие термины. 11p − 36> 7p − 2811p − 36> 7p − 28
    Вычтите 7p7p с обеих сторон, чтобы собрать
    переменных слева, поскольку 11> 7.11> 7.
    11п − 36−7p> 7p − 28−7p11p − 36−7p> 7p − 28−7p
    Упростить. 4p − 36> −284p − 36> −28
    Добавьте 36 к обеим сторонам, чтобы собрать константы
    справа.
    4p − 36 + 36> −28 + 364p − 36 + 36> −28 + 36
    Упростить. 4p> 84p> 8
    Разделим обе части неравенства на
    4; неравенство остается прежним.
    4p4> 844p4> 84
    Упростить. p> 2p> 2
    Изобразите решение на числовой прямой.
    Запишите решение в интервальной записи. (2, ∞) (2, ∞)

    Попробуйте 2.105

    Решите неравенство 9y + 2 (y + 6)> 5y − 249y + 2 (y + 6)> 5y − 24, обозначьте решение на числовой прямой и запишите решение в интервальной записи.

    Попробуйте 2.106

    Решите неравенство 6u + 8 (u − 1)> 10u + 326u + 8 (u − 1)> 10u + 32, обозначьте решение на числовой прямой и запишите решение в интервальной записи.

    Так же, как некоторые уравнения являются тождествами, а некоторые — противоречиями, неравенства могут быть тождествами или противоречиями. Мы узнаем эти формы, когда у нас остаются только константы при решении неравенства. Если результатом является истинное утверждение, у нас есть личность. Если результатом является ложное утверждение, мы приходим к противоречию.

    Пример 2.54

    Решите неравенство 8x − 2 (5 − x) <4 (x + 9) + 6x, 8x − 2 (5 − x) <4 (x + 9) + 6x, нанесите решение на числовой прямой и запишите решение в интервальной записи.

    Решение
    Максимально упростите каждую сторону. 8x − 2 (5 − x) <4 (x + 9) + 6x8x − 2 (5 − x) <4 (x + 9) + 6x
    Распространить. 8x − 10 + 2x <4x + 36 + 6x8x − 10 + 2x <4x + 36 + 6x
    Объедините похожие термины. 10x − 10 <10x + 3610x − 10 <10x + 36
    Вычтите 10 x с обеих сторон, чтобы собрать
    переменных слева.
    10x − 10−10x <10x + 36−10x10x − 10−10x <10x + 36−10x
    Упростить. −10 <36−10 <36
    Графики x исчезли, и у нас есть истинное утверждение
    .
    Неравенство — это идентичность.
    Решение — все действительные числа.
    Изобразите решение на числовой прямой.
    Запишите решение в интервальной записи. (−∞, ∞) (- ∞, ∞)

    Попробуйте 2.107

    Решите неравенство 4b − 3 (3 − b)> 5 (b − 6) + 2b4b − 3 (3 − b)> 5 (b − 6) + 2b, отобразите решение на числовой прямой и запишите решение. в интервальной записи.

    Попробуйте 2.108

    Решите неравенство 9h − 7 (2 − h) <8 (h + 11) + 8h9h − 7 (2 − h) <8 (h + 11) + 8h, отобразите решение на числовой прямой и запишите решение. в интервальной записи.

    Мы можем очистить дроби в неравенствах так же, как и в уравнениях.Опять же, будьте осторожны со знаками при умножении или делении на минус.

    Пример 2.55

    Решите неравенство 13a − 18a> 524a + 34,13a − 18a> 524a +34, обозначьте решение на числовой прямой и запишите решение в интервальной записи.

    Попробуйте 2.109

    Решите неравенство 14x − 112x> 16x + 7814x − 112x> 16x + 78, обозначьте решение на числовой прямой и запишите решение в интервальной записи.

    Попробуйте 2.110

    Решите неравенство 25z − 13z <115z −3525z − 13z <115z −35, нанесите решение на числовой прямой и запишите решение в интервальной записи.

    Переведите на неравенство и решите

    Чтобы перевести английские предложения в выражения неравенства, нам нужно распознавать фразы, указывающие на неравенство. Некоторые слова просты, например «больше чем» и «меньше чем». Но другие не так очевидны. В таблице 2.2 приведены некоторые общие фразы, указывающие на неравенство.

    >> ≥ ≥ << ≤≤
    больше

    больше

    больше

    больше
    больше или равно

    не менее

    не менее

    минимально
    меньше

    меньше

    меньше

    меньше
    меньше или равно

    не больше

    не больше

    максимум

    Таблица 2.2

    Пример 2.56

    Перевести и решить. Затем изобразите решение на числовой прямой и запишите решение в интервальной записи.

    Двадцать семь меньше xis не менее 48. Двадцать семь меньше xis не менее 48.

    Попробуйте 2.111

    Перевести и решить. Затем изобразите решение на числовой прямой и запишите решение в интервальной записи.

    Девятнадцать меньше р не меньше 47.

    Попробуйте 2.112

    Перевести и решить.Затем изобразите решение на числовой прямой и запишите решение в интервальной записи.

    На четыре больше a не больше 15.

    Решение приложений с линейными неравенствами

    Многие жизненные ситуации требуют от нас решения проблемы неравенства. Метод, который мы будем использовать для решения приложений с линейными неравенствами, очень похож на тот, который мы использовали при решении приложений с уравнениями.

    Мы прочитаем задачу и убедимся, что все слова понятны.Затем мы определим, что мы ищем, и назначим переменную для его представления. Мы сформулируем проблему в одном предложении, чтобы облегчить перевод в неравенство. Затем решим неравенство.

    Иногда приложение требует, чтобы решением было целое число, но алгебраическое решение неравенства не является целым числом. В этом случае мы должны округлить алгебраическое решение до целого числа. Контекст приложения будет определять, округлять ли мы в большую или меньшую сторону.

    Пример 2.57

    Доун выиграла мини-грант в размере 4000 долларов на покупку планшетных компьютеров для своего класса. Планшеты, которые она хотела бы купить, стоят 254,12 доллара каждый, включая налоги и доставку. Какое максимальное количество планшетов может купить Dawn?

    Решение
    Шаг 1. Прочтите проблему.
    Шаг 2. Определите , что вы ищете. максимальное количество планшетов Dawn можно купить
    Шаг 3.Назовите то, что вы ищете.
    Выберите переменную для представления этого количества. Letn = количество таблеток Letn = количество таблеток.
    Шаг 4. Переведите . Напишите предложение, которое дает информацию, чтобы найти его. $ 254,12 умножить на количество планшетов не более 4000 долларов.
    Перевести в неравенство. 254,12n≤4000 254,12n≤4000
    Шаг 5.Решите неравенство.
    Но n должно быть целым числом таблеток, поэтому округляем до 15.
    n≤15.74n≤15n≤15.74n≤15
    Шаг 6. Проверьте ответ в проблеме и убедитесь, что он имеет смысл.
    Округляя цену до 250 долларов, 15 планшетов будут стоить 3750 долларов, а 16 планшетов — 4000 долларов. Так что максимум 15 планшетов по цене 254,12 доллара кажется разумным.
    Шаг 7.Ответьте на вопрос полным предложением. Dawn можно купить максимум 15 таблеток.

    Попробуйте 2.113

    У Энджи есть 20 долларов, которые она может потратить на коробки сока для дошкольного пикника сына. Каждая упаковка коробок для сока стоит 2,63 доллара. Какое максимальное количество пакетов она может купить?

    Попробуйте 2.114

    Дэниел хочет удивить свою девушку днём рождения в её любимом ресторане. Ужин будет стоить 42,75 доллара на человека, включая чаевые и налог.Его бюджет на вечеринку составляет 500 долларов. Какое максимальное количество людей может присутствовать на вечеринке Дэниел?

    Пример 2.58

    Телефонный план

    Талейши стоит 28,80 доллара в месяц плюс 0,20 доллара за текстовое сообщение. Сколько текстовых сообщений она может отправлять / получать, при этом ежемесячный счет за телефонные разговоры не превышает 50 долларов?

    Решение
    Шаг 1. Прочтите проблему.
    Шаг 2. Определите , что вы ищете. количество текстовых сообщений Талейша может составить
    Шаг 3. Назовите то, что вы ищете.
    Выберите переменную для представления этого количества. Lett = количество текстовых сообщений Lett = количество текстовых сообщений.
    Шаг 4. Переведите Напишите предложение, которое дает информацию, чтобы найти его. 28,80 доллара плюс 0,20 доллара, умноженное на количество текстовых сообщений, меньше или равно 50 долларам.
    Перевести в неравенство. 28.80 + 0.20t≤50 28.80 + 0.20t≤50
    Шаг 5. Решите неравенство . 0,2t≤21,2t≤106 текстовых сообщений 0,2t≤21,2t≤106 текстовых сообщений
    Шаг 6. Проверьте ответ в проблеме и убедитесь, что он имеет смысл.
    Да, 28,80 + 0,20 (106) = 50. Да, 28,80 + 0,20 (106) = 50.
    Шаг 7. Напишите предложение, которое отвечает на вопрос. Талейша может отправлять / получать не более 106 текстовых сообщений, чтобы ее счет не превышал 50 долларов.

    Попробуйте 2.115

    У Серджио и Лизет очень ограниченный бюджет на отпуск. Они планируют арендовать автомобиль у компании, которая взимает 75 долларов в неделю плюс 0,25 доллара за милю. Сколько миль они могут проехать в течение недели, не выходя за рамки своего бюджета в 200 долларов?

    Попробуйте 2.116

    Счет за отопление

    Rameen составляет 5,42 доллара в месяц плюс 1,08 доллара за терм.Сколько термосов может использовать Рамин, если он хочет, чтобы его счет за отопление составлял не более 87,50 долларов.

    Прибыль — это деньги, которые остаются после вычета затрат из выручки. В следующем примере мы найдем количество работ, которые маленькая бизнес-леди должна выполнять каждый месяц, чтобы получать определенную прибыль.

    Пример 2.59

    Фелисити занимается каллиграфией. Она берет 2,50 доллара за приглашение на свадьбу. Ее ежемесячные расходы составляют 650 долларов. Сколько приглашений она должна написать, чтобы получать прибыль не менее 2800 долларов в месяц?

    Решение
    Шаг 1.Прочтите проблему.
    Шаг 2. Определите , что вы ищете. количество приглашений, которые нужно написать Фелисити
    Шаг 3. Назовите то, что вы ищете.
    Выберите переменную для ее представления.
    Letj = количество приглашений Letj = количество приглашений.
    Шаг 4. Перевести. Напишите предложение, которое дает информацию, чтобы найти его. 2,50 доллара США, умноженное на количество приглашений минус 650 долларов, составляет не менее 2800 долларов.
    Перевести в неравенство. 2,50j-650≥2,8002,50j-650≥2,800
    Шаг 5. Решите неравенство . 2,5j≥3,450j≥1,380призваний2,5j≥3,450j≥1,380призваний
    Шаг 6. Проверьте ответ в проблеме и убедитесь, что он имеет смысл.
    Если бы Фелисити написала 1400 приглашений, ее прибыль составила бы
    2.50 (1400) — 650, или 2850 долларов. Это больше 2800 долларов.
    Шаг 7. Напишите предложение, которое отвечает на вопрос. Фелисити должна написать не менее 1380 приглашений.

    Попробуйте 2.117

    Калеб занимается присмотром за домашними животными. Он берет 32 доллара в час. Его ежемесячные расходы составляют 2272 доллара. Сколько часов он должен работать, чтобы получать прибыль не менее 800 долларов в месяц?

    Попробуйте 2.118

    Elliot занимается обслуживанием ландшафтов.Его ежемесячные расходы составляют 1100 долларов. Если он берет 60 долларов за работу, сколько работ он должен сделать, чтобы получать прибыль не менее 4000 долларов в месяц?

    Есть много ситуаций, когда несколько количеств вносят вклад в общие расходы. Когда мы решаем подобные проблемы, мы должны учитывать все индивидуальные расходы.

    Пример 2.60

    Малик планирует шестидневную поездку на летние каникулы. У него есть сбережения в размере 840 долларов, и он зарабатывает 45 долларов в час за репетиторство. Поездка обойдется ему в 525 долларов на авиабилеты, 780 долларов на еду и осмотр достопримечательностей и 95 долларов за ночь в отеле.Сколько часов он должен заниматься репетитором, чтобы хватило денег на поездку?

    Решение
    Шаг 1. Прочтите проблему.
    Шаг 2. Определите , что вы ищете. количество часов, которые Малик должен преподавать
    Шаг 3. Назовите то, что вы ищете.
    Выберите переменную для представления этого количества. Leth = количество часов Leth = количество часов.
    Шаг 4. Перевести. Напишите предложение, которое дает информацию, чтобы найти его. Расходы должны быть меньше или равны доходу. Стоимость авиабилета плюс стоимость еды и осмотра достопримечательностей, а также счет в отеле должны быть меньше суммы сбережений плюс сумма заработанного репетиторства.
    Перевести в неравенство. 525 + 780 + 95 (6) ≤840 + 45х525 + 780 + 95 (6) ≤840 + 45х
    Шаг 5.Решите неравенство. 1,875≤840 + 45h2,035≤45h33≤hh≥231,875≤840 + 45h2,035≤45h33≤hh≥23
    Шаг 6. Проверьте ответ в проблеме и убедитесь, что он имеет смысл.
    Подставляем 23 в неравенство.
    1,875≤840 + 45h2,875≤840 + 45 (23) 1,875≤1875 1,875≤840 + 45h2,875≤840 + 45 (23) 1,875≤1875
    Шаг 7. Напишите предложение, которое отвечает на вопрос. Малик должен быть репетитором не менее 23 часов.

    Попробуйте 2.119

    У лучшей подруги Бренды свадьба по назначению, мероприятие продлится три дня. У Бренды 500 долларов сбережений, и она может зарабатывать 15 долларов в час за присмотром за детьми. Она рассчитывает заплатить 350 долларов за авиабилеты, 375 долларов за еду и развлечения и 60 долларов за ночь за свою долю гостиничного номера. Сколько часов она должна сидеть с ребенком, чтобы иметь достаточно денег, чтобы оплатить поездку?

    Попробуйте 2.120

    Хосуэ хочет отправиться в путешествие с друзьями на 10 ночей следующей весной.Ему будет стоить 180 долларов на бензин, 450 долларов на еду и 49 долларов за ночь в номере мотеля. У него 520 долларов сбережений, и он может заработать 30 долларов за уборку снега на проезжей части. Сколько проездов он должен прорыть, чтобы иметь достаточно денег, чтобы заплатить за поездку?

    Раздел 2.5 Упражнения

    Практика ведет к совершенству

    Неравенства в графике на числовой прямой

    В следующих упражнениях нарисуйте каждое неравенство на числовой прямой и запишите в интервальной нотации.

    296.


    ⓐ x <−2x <−2
    ⓑ x≥ − 3,5x≥ − 3,5
    ⓒ x≤23x≤23

    297.


    x> 3x> 3
    ⓑ x≤ − 0,5x≤ − 0,5
    ⓒ x≥13x≥13

    298.


    x≥ − 4x≥ − 4
    ⓑ x <2,5x <2,5
    ⓒ x> −32x> −32

    299.


    ⓐ x≤5x≤5
    ⓑ x≥ − 1,5x≥ − 1,5
    ⓒ x <−73x <−73

    300.


    ⓐ −5 ⓑ −3≤x <1−3≤x <1
    ⓒ 0≤x≤1,50≤x≤1,5

    301.


    ⓐ −2 ⓑ −5≤x <−3−5≤x <−3
    ⓒ 0≤x≤3.50≤x≤3,5

    302.


    ⓐ −1 ⓑ −3 ⓒ −1,25≤x≤0−1,25≤x≤0

    303.


    ⓐ −4 ⓑ −5 ⓒ −3,75≤x≤0−3,75≤x≤0

    Решите линейные неравенства

    В следующих упражнениях решите каждое неравенство, нанесите решение на числовой прямой и запишите решение в интервальной записи.

    304.


    ⓐ a + 34≥710a + 34≥710
    ⓑ 8x> 728x> 72
    ⓒ 20> 25h30> 25h

    305.


    ⓐ b + 78≥16b + 78≥16
    ⓑ 6y <486y <48
    ⓒ 40 <58k40 <58k

    306.


    ⓐ f − 1320 <−512f − 1320 <−512
    ⓑ 9t≥ − 279t≥ − 27
    ⓒ 76j≥4276j≥42

    307.


    ⓐ г − 1112 <−518 г − 1112 <−518
    ⓑ 7 с <−287 с <−28
    ⓒ 94g≤3694g≤36

    308.


    ⓐ −5u≥65−5u≥65
    ⓑ a − 3≤9a − 3≤9

    309.


    ⓐ −8v≤96−8v≤96
    ⓑ b − 10≥30b − 10≥30

    310.


    ⓐ −9c <126−9c <126
    ⓑ −25

    311.


    ⓐ −7d> 105−7d> 105
    ⓑ −18> q − 6−18> q − 6

    В следующих упражнениях решите каждое неравенство, нанесите решение на числовой прямой и запишите решение в интервальной записи.

    316.

    12x + 3 (x + 7)> 10x − 2412x + 3 (x + 7)> 10x − 24

    317.

    9y + 5 (y + 3) <4y − 359y + 5 (y + 3) <4y − 35

    318.

    6h − 4 (час − 1) ≤7h − 116h − 4 (час − 1) ≤7h − 11

    319.

    4k− (k − 2) ≥7k − 264k- (k − 2) ≥7k − 26

    320.

    8 м − 2 (14 − м) ≥ 7 (м − 4) + 3 м 8 м − 2 (14 − м) ≥ 7 (м − 4) + 3 м

    321.

    6n − 12 (3 − n) ≤9 (n − 4) + 9n6n − 12 (3 − n) ≤9 (n − 4) + 9n

    322.

    34b-13b <512b-1234b-13b <512b-12

    323.

    9u + 5 (2u − 5) ≥12 (u − 1) + 7u9u + 5 (2u − 5) ≥12 (u − 1) + 7u

    324.

    23 г-12 (г-14) ≤16 (г + 42) 23 г-12 (г-14) ≤16 (г + 42)

    325.

    45h − 23 (h − 9) ≥115 (2h + 90) 45h − 23 (h − 9) ≥115 (2h + 90)

    326.

    56a − 14a> 712a + 2356a − 14a> 712a + 23

    327.

    12v + 3 (4v − 1) ≤19 (v − 2) + 5v12v + 3 (4v − 1) ≤19 (v − 2) + 5v

    В следующих упражнениях решите каждое неравенство, нанесите решение на числовой прямой и запишите решение в интервальной записи.

    330.

    23p − 2 (6−5p)> 3 (11p − 4) 23p − 2 (6−5p)> 3 (11p − 4)

    331.

    18q − 4 (10−3q) <5 (6q − 8) 18q − 4 (10−3q) <5 (6q − 8)

    332.

    −94x≥ − 512−94x≥ − 512

    333.

    −218y≤ − 1528−218y≤ − 1528

    Перевести на неравенство и разрешить

    В следующих упражнениях переведите и решите. Затем изобразите решение на числовой прямой и запишите решение в интервальной записи.

    338.

    Три больше ч не меньше 25.

    339.

    Шесть больше k больше 25.

    340.

    Ten меньше w не меньше 39.

    341.

    Двенадцать меньше x не меньше 21.

    342.

    Отрицательно пять раз р не больше 95.

    343.

    Дважды отрицательное значение с меньше 56.

    344.

    Девятнадцать меньше b не больше -22,22.

    345.

    Пятнадцать меньше , а не меньше −7.−7.

    Решение приложений с линейными неравенствами

    В следующих упражнениях решите.

    346.

    Алан загружает поддон ящиками, каждая из которых весит 45 фунтов. Поддон может безопасно выдержать не более 900 фунтов. Сколько ящиков он может безопасно загрузить на поддон?

    347.

    На лифте в многоквартирном доме Йехира есть табличка, на которой указано, что максимальный вес составляет 2100 фунтов. Если средний вес одного человека составляет 150 фунтов, сколько людей могут безопасно пользоваться лифтом?

    348.

    Андре просматривает апартаменты с тремя своими друзьями. Они хотят, чтобы ежемесячная арендная плата не превышала 2360 долларов. Если соседи по комнате поровну распределяют арендную плату между четырьмя из них, какова максимальная арендная плата, которую каждый будет платить?

    349.

    Арлин получила подарочную карту на 20 долларов в кофейню. Ее любимый напиток со льдом стоит 3,79 доллара. Какое максимальное количество напитков она может купить по подарочной карте?

    350.

    Тиган любит играть в гольф. В следующем месяце он заложил 60 долларов на тренировочное поле.Каждый раз, когда он идет, ведро с мячиками обходится ему в 10,55 доллара. Какое максимальное количество раз он может посещать тренировочное поле в следующем месяце?

    351.

    Райан взимает со своих соседей 17,50 долларов за мытье машины. Сколько машин он должен помыть следующим летом, если его цель — заработать не менее 1500 долларов?

    352.

    Кешад получает 2400 долларов в месяц плюс 6% от его продаж. Его брат зарабатывает 3300 долларов в месяц. На какой общий объем продаж ежемесячная зарплата Кешада будет выше, чем месячная зарплата его брата?

    353.

    Кимуен нужно зарабатывать 4150 долларов в месяц, чтобы оплачивать все свои расходы. Ее работа приносит ей 3 475 долларов в месяц плюс 4% от общего объема продаж. Каков минимальный общий объем продаж Кимуен, чтобы она могла оплатить все свои расходы?

    354.

    Андре предложили работу начального уровня. Компания предлагала ему 48 000 долларов в год плюс 3,5% от его общих продаж. Андре знает, что средняя заработная плата за эту работу составляет 62 000 долларов. Каким должен быть общий объем продаж Андре, чтобы его зарплата была не меньше средней заработной платы за эту работу?

    355.

    Наталья рассматривает два предложения о работе. На первой работе ей платили 83 тысячи долларов в год. Второй заплатит ей 66 500 долларов плюс 15% от общего объема продаж. Каким должен быть ее общий объем продаж, чтобы ее зарплата по второму предложению была выше, чем по первому?

    356.

    Счет за воду Джейка составляет 24,80 доллара в месяц плюс 2,20 доллара за кубический фут (сто кубических футов) воды. Какое максимальное количество ccf может использовать Джейк, если он хочет, чтобы его счет не превышал 60 долларов?

    357. Телефонный тариф

    Киёси стоит 17 долларов.50 в месяц плюс 0,15 доллара за текстовое сообщение. Какое максимальное количество текстовых сообщений может использовать Киёси, чтобы телефонный счет не превышал 56,60 доллара?

    358. Тарифный план

    Марлона стоит 49,99 доллара в месяц плюс 5,49 доллара за первый просмотр фильма. Сколько фильмов он сможет посмотреть в первый раз, если хочет, чтобы его ежемесячный счет составлял не более 100 долларов?

    359.

    Келлен хочет снять банкетный зал в ресторане для детского душа своей кузины. Ресторан стоит 350 долларов за банкетный зал плюс 32 доллара.50 на человека за обед. Сколько людей может принять душ Келлен, если она хочет, чтобы максимальная стоимость была 1500 долларов?

    360.

    Мошде ведет парикмахерский бизнес из своего дома. Она берет 45 долларов за стрижку и укладку. Ее ежемесячные расходы составляют 960 долларов. Она хочет иметь возможность вкладывать не менее 1200 долларов в месяц на свой сберегательный счет, чтобы открыть собственный салон. Сколько «стилей и стилей» ей нужно сделать, чтобы сэкономить не менее 1200 долларов в месяц?

    361.

    Ной устанавливает и настраивает программное обеспечение на домашних компьютерах.Он берет 125 долларов за работу. Его ежемесячные расходы составляют 1600 долларов. Сколько рабочих мест он должен выполнить, чтобы получить прибыль не менее 2400 долларов?

    362.

    Кэтрин — личный повар. Она берет 115 долларов за обед на четырех человек. Ее ежемесячные расходы составляют 3150 долларов. Сколько обедов для четырех человек она должна продать, чтобы получить прибыль не менее 1900 долларов?

    363.

    Мелисса делает ожерелья и продает их в Интернете. Она берет 88 долларов за ожерелье. Ее ежемесячные расходы составляют 3745 долларов. Сколько ожерелий она должна продать, если хочет получить прибыль не менее 1650 долларов?

    364.

    Пятеро чиновников студенческого самоуправления хотят пойти на съезд штата. Это будет стоить им 110 долларов за регистрацию, 375 долларов на транспорт и еду и 42 доллара на человека в отеле. На сберегательный счет студенческого самоуправления заложено 450 долларов на съезд. Остальные деньги они могут заработать на мойке автомобилей. Если они берут 5 долларов за машину, сколько машин они должны помыть, чтобы иметь достаточно денег для оплаты поездки?

    365.

    Сезар планирует четырехдневную поездку, чтобы навестить своего друга в колледже в другом штате.Это будет стоить ему 198 долларов на авиабилеты, 56 долларов на местный транспорт и 45 долларов в день на еду. У него 189 долларов сбережений, и он может заработать по 35 долларов за каждую стриженную лужайку. Сколько газонов нужно косить, чтобы на поездку хватило денег?

    366.

    Алонзо работает мастером по ремонту автомобилей. Он берет 175 долларов за машину. Он планирует переехать из родительского дома и снять свою первую квартиру. Ему нужно будет заплатить 120 долларов за подачу заявления, 950 долларов за залог, а также арендную плату за первый и последний месяц из расчета 1140 долларов в месяц.У него 1810 долларов сбережений. Сколько машин нужно собрать, чтобы иметь достаточно денег на аренду квартиры?

    367.

    Ын-Кён работает репетитором и зарабатывает 60 долларов в час. У нее 792 доллара сбережений. Она планирует отпраздновать годовщину своих родителей. Она хочет пригласить 40 гостей. Вечеринка обойдется ей в 1520 долларов на еду и напитки и 150 долларов на фотографа. Она также окажет услугу каждому из гостей, и каждая услуга будет стоить 7,50 долларов. Сколько часов она должна заниматься репетитором, чтобы денег на вечеринку хватило?

    Повседневная математика
    368.

    Максимальная нагрузка на сцену В 2014 году обрушилась сцена средней школы в Фуллертоне, Калифорния, когда 250 учеников вышли на сцену для финала музыкальной постановки. Пострадали два десятка студентов. Сцена могла выдержать максимум 12 750 фунтов. Если предполагается, что средний вес студента составляет 140 фунтов, каково максимальное количество студентов, которые могут безопасно выйти на сцену?

    369.

    Максимальный вес лодки В 2004 году в гавани Балтимора затонуло водное такси, пять человек утонули.Водное такси имело максимальную вместимость 3500 фунтов (25 человек при среднем весе 140 фунтов). Средний вес 25 человек в водном такси, когда оно затонуло, составлял 168 фунтов на человека. Каким должно быть максимальное количество людей с таким весом?

    370.

    Свадебный бюджет Адель и Уолтер нашли идеальное место для своего свадебного приема. Стоимость составляет 9850 долларов США на 100 гостей, плюс 38 долларов США на каждого дополнительного гостя. Сколько гостей может присутствовать, если Адель и Уолтер хотят, чтобы общая стоимость не превышала 12500 долларов?

    371.

    Бюджет душа Пенни планирует детский душ для своей невестки. Стоимость ресторана составляет 950 долларов США за 25 гостей, плюс 31,95 доллара США за каждого дополнительного гостя. Сколько гостей может присутствовать, если Пенни хочет, чтобы общая стоимость не превышала 1500 долларов?

    Письменные упражнения
    372.

    Объясните, почему необходимо обратить неравенство при решении −5x> 10. −5x> 10.

    373.

    Объясните, почему необходимо обратить неравенство при решении n − 3 <12.п-3 <12.

    374.

    Найдите свой телефонный счет за последний месяц и почасовую зарплату, которую вам платят на работе. Подсчитайте количество часов работы, которое вам потребуется, чтобы заработать хотя бы достаточно денег для оплаты телефонного счета, написав соответствующее неравенство и затем решив его. Считаете ли вы, что это подходящее количество часов? Подходит ли вам этот тарифный план?

    375.

    Узнайте, сколько единиц у вас осталось после этого семестра для достижения вашей цели в колледже, и оцените количество единиц, которое вы можете сдавать в каждом семестре в колледже.Подсчитайте количество терминов, которое потребуется вам для достижения цели в колледже, написав соответствующее неравенство и затем решив его. Это приемлемое количество терминов, пока вы не достигнете своей цели? Как бы вы могли ускорить этот процесс?

    Самопроверка

    ⓐ После выполнения упражнений используйте этот контрольный список, чтобы оценить свое мастерство в достижении целей этого раздела.

    ⓑ После просмотра контрольного списка, думаете ли вы, что хорошо подготовились к следующему разделу? Почему или почему нет?

    Решение рациональных неравенств — ChiliMath

    Ключевой подход к решению рациональных неравенств основан на поиске критических значений рационального выражения, которые делят числовую прямую на отдельные открытые интервалы.

    Критические значения — это просто нули числителя и знаменателя. Вы должны помнить, что нули знаменателя делают рациональное выражение неопределенным, поэтому их следует немедленно игнорировать или исключать как возможное решение. Однако нули числителя также необходимо проверять на предмет его возможного включения в общее решение.

    В этом уроке я рассмотрю пять (5) рабочих примеров с разным уровнем сложности, чтобы проиллюстрировать как процедуры, так и концепции.


    Примеры решения рациональных неравенств

    Пример 1: Решите приведенное ниже рациональное неравенство.

    Я начинаю решать это рациональное неравенство с написания его в общем виде. Общая форма подразумевает, что рациональное выражение расположено слева от неравенства, а ноль остается справа.

    Общая форма имеет четыре (4) типа.

    Приятно знать, что эта проблема уже в общем виде.Мой следующий шаг — найти нулей числителя и знаменателя .

    Я могу найти нули числителя, полностью вычленив его, а затем отдельно установить каждый множитель равным нулю и решить относительно x. Точно так же поиск нулей знаменателя выполняется таким же образом.

    Теперь я буду использовать нули, чтобы разделить числовую строку на интервалы. Нули числителя и знаменателя также известны как критические числа . В этом случае два критических числа делят числовую прямую на три отдельных интервала.

    Следующий шаг — выбрать или выбрать число в каждом интервале и вычислить его обратно в исходное рациональное неравенство; чтобы определить, истинное это утверждение или ложное. Истинное утверждение означает, что интервал является частью решения, в противном случае это не так.

    Как видите, числа, которые я выбрал для каждого интервала, выделены желтым.

    Обратите внимание, что открытый интервал между -1 и 3, записанный как \ left ({- 1,3} \ right), дает истинное утверждение, которое подразумевает, что оно является частью решения.

    Итак, где еще нам искать возможные решения, чтобы покончить с этим?

    Проверяйте нули или критические числа числителей только в исходном уравнении. Если это верно, включите это критическое число как часть общего решения.

    Нулей в числителе 3. Сейчас проверю.

    Использование квадратной скобки указывает, что это часть решения, а открытая скобка (скобка) означает, что это не так.Я напишу свой окончательный ответ как \ left ({- 1, \ left. 3 \ right]} \ right ..


    Пример 2: Решите приведенное ниже рациональное неравенство.

    Во-первых, данное рациональное неравенство имеет общий вид, потому что рациональное выражение находится слева, а ноль — справа. Это хорошо!

    Затем я вынесу за скобки числитель и знаменатель. После этого у вас должно получиться что-то вроде этого.

    Теперь я могу найти нули числителя и знаменателя.

    Эти нули или критические числа делят числовую строку на отдельные интервалы или части.

    Выберите номер теста для каждого интервала и верните исходное рациональное неравенство.

    Используйте факторизованную форму исходного рационального неравенства , чтобы оценить числа тестов для упрощения вычислений.

    Цифры желтого цвета — это те числа, которые я выбрал для проверки достоверности каждого интервала.

    Интервалы, дающие истинные утверждения:

    Чтобы найти остальную часть решения, проверьте правильность нулей числителя только в исходном рациональном неравенстве.

    Если вы сделали это правильно, вы должны согласиться с тем, что −4 и 2 — это недействительные ответы , потому что они не дают истинных утверждений после проверки.

    Окончательный ответ на эту проблему в обозначении интервалов —


    Пример 3: Решите приведенное ниже рациональное неравенство.

    Я бы сначала вычленил числитель и знаменатель, чтобы найти их нули. В факторизованном виде у меня получилось

    Затем определите нули рационального неравенства, установив каждый коэффициент равным нулю, а затем решив относительно x.

    • Нули числителя: –1 и 4
    • Нули знаменателя: 4

    Используйте нули как критические числа, чтобы разделить числовую прямую на отдельные интервалы. Я начинаю проверять достоверность каждого интервала, выбирая тестовые значения и оценивая их в соответствии с исходным рациональным неравенством. Желтым — числа, которые я выбрал.

    Обратите внимание, что единственный интервал, дающий истинное утверждение, — это \ left ({- 1,4} \ right).

    Более того, нули числителя не совпадают с исходным рациональным неравенством, поэтому я должен их игнорировать.

    Окончательный ответ — просто \ left ({- 1,4} \ right).


    Пример 4: Решите приведенное ниже рациональное неравенство.

    Это рациональное неравенство не в общем виде . Правая часть должна быть равна нулю. Первый шаг — избавиться от константы на этой стороне путем вычитания обеих частей на 1. После этого сведите к одному рациональному выражению. У вас должен быть аналогичный предварительный шаг, вот так.

    Затем найдите нули числителя и знаменателя.

    • Нули в числителе: -7
    • Нули в знаменателе: -3

    Используйте нули как критические числа, чтобы разделить числовую линию на участки или интервалы.

    Затем выберите номера тестов для каждого интервала и оцените их в общей форме, чтобы определить их истинностные значения. Желтым цветом обозначены выбранные значения. Вы можете выбрать другие числа, если они находятся в проверяемом интервале.

    Интервалы, дающие истинные утверждения:

    Между тем, после проверки нуля числителя при x = — \, 7 это также приводит к истинному утверждению.Используйте квадратную скобку, чтобы указать, что он включен в качестве решения.

    Окончательный ответ в обозначении интервалов должен быть


    Пример 5: Решите приведенное ниже рациональное неравенство.

    Мне нужно обнулить правую часть рационального неравенства. Для этого я буду одновременно прибавлять x и вычитать 5 с обеих сторон. Однако моя конечная цель — выразить это в едином рациональном выражении. Здесь вам пригодятся ваши навыки складывания и вычитания рациональных выражений.У вас должны быть аналогичные шаги ниже.

    Затем найдите нули числителя и знаменателя.

    • Нули числителя: -3 и 5
    • Нули знаменателя: 0

    Используйте нули, чтобы разделить числовую прямую на отдельные интервалы. Выберите номера тестов для каждого интервала, чтобы проверить, верны ли утверждения. Выбранные тестовые значения для x выделены желтым цветом.

    «Истинные» интервалы — это \ left ({- \, \ infty, — 3} \ right) и \ left ({0,5} \ right).Более того, нули числителя также сверяются с общим видом данного рационального неравенства. Следовательно, я должен включить -3 и 5 как часть решения с использованием квадратных скобок.

    Окончательный ответ теперь становится


    Практика с рабочими листами

    Возможно, вас заинтересует:

    Решение рациональных уравнений

    Сложение и вычитание рациональных выражений

    Умножение рациональных выражений

    Неравенство

    Неравенство

    Содержание : Эта страница соответствует § 2.5 (стр. 216) текст.

    Предлагаемые задачи из текста:

    с. 225 # 11, 12, 13, 14, 16, 28, 33, 35, 38, 41, 53, 56, 62, 63, 68, 69

    Линейные неравенства

    Сочетания неравенств

    Неравенства, связанные с абсолютными значениями

    Полиномиальные неравенства

    Рациональные неравенства


    Линейные неравенства

    Неравенство — это сравнение выражений на «меньше» (<), «меньше или равно» на «(<=)», «больше» (>) или «больше или равно» (> =).Обратите внимание, что Html не поддерживает стандартные символы «меньше или равно» и «больше или равно», поэтому мы используем <= и> = для этих отношений.

    Пример 1 . х + 3 <= 10

    Решение для неравенства в x — это число, такое, что когда мы подставляем это число вместо x, мы имеем верное заявление. Итак, 4 — это решение, например 1, а 8 — нет. Набор решений неравенства: набор всех решений.Обычно неравенство имеет бесконечно много решений, и множество решений легко описывается с использованием обозначения интервалов.

    Набор решений в примере 1 — это набор всех x <= 7. В интервальной записи это набор (-inf, 7], где мы используем inf для обозначения бесконечности.

    Линейное неравенство — это такое неравенство, что если бы мы заменили неравенство соотношением равенства, то мы имели бы линейное уравнение. Решение линейных неравенств очень похоже на решение линейных уравнений с одним важным отличием.

    Когда вы умножаете или делите обе стороны неравенства на отрицательное число, направление неравенство обращено вспять.

    Вы можете увидеть это, используя неравенство без переменных.

    Пример 2 .

    3 <7. Это ИСТИНА.

    (3) (- 2) <(7) (- 2). Это ЛОЖЬ, потому что -6 находится справа от -14 в числовой строке.Следовательно, -6> -14.

    (3) (- 2)> (7) (- 2). Это верно. Итак, когда мы умножаем исходное неравенство на -2, мы должны перевернуть направление, чтобы получить еще одно верное утверждение.

    Примечание : Как правило, мы не можем умножать или делить обе стороны неравенства на выражение с переменной, потому что некоторые значения переменной могут сделать выражение положительным, а некоторые — отрицательным.

    Пример 3 .

    7 — 2x <3.

    -2x <-4.

    x> 2.

    Примечание : Когда мы разделили обе стороны неравенства на -2, мы изменили направление неравенства.

    Посмотрите на графики функций по обе стороны от неравенства.

    Для выполнения неравенства 7 — 2x должно быть меньше 3. Итак, мы ищем такие числа x, что точка на графике y = 7 — 2x находится на ниже точки на графике y = 3.Это верно для x> 2. В интервале обозначение множество решений — (2, inf).

    Есть еще один способ использовать графическую утилиту для решения этого неравенства. В Java Grapher выражение (7-2 * x) L3 имеет значение 1 для чисел x, удовлетворяющих неравенству, и значение 0 для других чисел Икс. На рисунке ниже показан график (7-2 * x) L3, нарисованный Grapher.

    Упражнение 1:

    Решите неравенство 4 — x> 1 + 3x.Ответ

    Вернуться к содержанию

    Сочетания неравенств

    Пример 4 .

    Найдите все числа x такие, что -3 <5 - 2x и 5 — 2x <9.

    -3 <5-2x

    -8 <-2x

    4> x

    (-inf, 4)

    И

    5 — 2x <9

    -2x <4

    x> -2

    (-2, инф)

    Чтобы удовлетворить обоим неравенствам, число должно быть в обоих наборах решений.Итак, числа, удовлетворяющие обоим неравенства — это значения на пересечении двух наборов решений, которые представляют собой набор (-2, 4) в интервале обозначение.

    Задача выше обычно записывается как двойное неравенство .

    -3 <5 - 2x <9 означает -3 <5 - 2x и 5 — 2x <9.

    Примечание: Когда мы решили два неравенства по отдельности, шаги в двух задачах были одно и тоже.Следовательно, для одновременного решения неравенств можно использовать обозначение двойного неравенства.

    -3 <5 - 2x <9.

    -8 <-2x <4.

    4> х> -2.

    В терминах графиков эта задача соответствует нахождению таких значений x, что соответствующая точка на график y = 5 — 2x находится между графиками y = -3 и y = 9.

    Пример 5 .

    Найдите все числа x такие, что x + 1 <0 или x + 1> 3.

    В примере 4 выше мы искали числа, удовлетворяющие обоим неравенствам. Здесь мы хотим найти числа удовлетворяющие любому из неравенств. Это соответствует объединению наборов решений вместо пересечения.

    Не используйте в этой ситуации обозначение двойного неравенства.

    x + 1 <0

    x <-1

    (-inf, -1)

    ИЛИ x + 1> 3

    x> 2

    (2, инф.)

    Набор решений представляет собой объединение двух интервалов (-inf, -1) и (2, inf).

    Упражнение 2:

    (a) 1 <3 + 5x <7 Ответ

    (b) 2 — x <1 или 2 - x> 5 Ответ

    Вернуться к содержанию

    Неравенства, связанные с абсолютными значениями

    Неравенства, включающие абсолютные значения, можно переписать как комбинации неравенств.

    Пусть a будет положительным числом.

    Чтобы понять эти утверждения, подумайте о числовой прямой. Абсолютное значение числа — это расстояние номер начинается с 0 в числовой строке. Итак, неравенство | x |

    Неравенство | x | > a удовлетворяют числам, расстояние от которых до 0 больше, чем a.Это означает числа которые либо больше, либо меньше -a.

    Пример 6 .

    | 3 + 2x | <= 7.

    -7 <= 3 + 2x <= 7.

    -10 <= 2x <= 4.

    -5 <= x <= 2.

    x находится в [-5, 2].

    Что касается графиков, мы ищем такие значения x, что соответствующая точка на графике y = | 3+ 2x | либо ниже, либо равна точке на графике y = 7.

    Пример 7 .

    | 5 — 2x | > 3.

    5 — 2x <-3 или 5 - 2x> 3.

    -2x <-8 или -2x> -2.

    x> 4 или x <1.

    x находится в (4, inf) union (-inf, 1).

    Этот набор решений соответствует области, где график y = | 5 — 2x | находится над графиком y = 3.

    Упражнение 3 :

    Решите следующие неравенства.Используйте графическую утилиту, чтобы проверить свои ответы.

    (а) | 3 + х | <4.

    (б) | 2 — х | > 3.

    Вернуться к содержанию

    Полиномиальные неравенства

    Пример 8 .

    x 2 — x — 6 <0.

    Первый шаг — найти нули многочлена x 2 — x — 6.

    х 2 — х — 6 = 0.

    (x + 2) (x — 3) = 0.

    x = -2 или x = 3.

    -2 и 3 называются критическими числами неравенства.

    Примечание: -2 и 3 не входят в набор решений неравенства. Мы ищем ценности x, где многочлен отрицательный. Множество решений неравенства соответствует области, в которой граф полинома ниже оси абсцисс. Критические числа -2 и 3 — это места пересечения графика ось абсцисс.

    Критические числа делят ось абсцисс на три интервала, называемых контрольными интервалами для неравенства.

    Интервалы тестирования: (-inf, -2), (-2, 3), (3, inf).

    Мы собираемся использовать тот факт, что полиномиальные функции непрерывны . Это означает, что их графики не делать перерывов и прыжков.

    Так как мы нашли все пересечения по оси x графика x 2 — x — 6 на протяжении каждого тестового интервала график должен быть либо выше оси x, либо ниже нее.Здесь нам нужно знать, что на графике нет никаких перерывов. Это означает, что мы можем выбрать любое число, которое нам нравится в тестовом интервале, и оценить полином на это число, чтобы увидеть, находится ли график выше или ниже оси x на протяжении всего интервала тестирования.

    (-inf, -2): -5 находится в интервале. (-5) 2 — (-5) — 6 = 24> 0, поэтому график y = x 2 — x — 6 находится над осью x на всем интервале (-inf, -2).

    (-2; 3): 0 находится в интервале.0 2 -0 — 6 = -6 <0, поэтому график y = x 2 — x — 6 находится ниже оси абсцисс на всем интервале.

    (3, inf): 4 находится в интервале. 4 2 -4-6 = 6> 0, поэтому график y = x 2 — x — 6 находится над осью абсцисс на всем интервале.

    Поскольку мы ищем области, в которых график находится ниже оси, набор решений равен -2

    Распространенная ошибка

    Мы будем использовать проблему из примера 8, чтобы проиллюстрировать распространенную ошибку.

    x 2 — x — 6 <0.

    (x + 2) (x — 3) <0 Хорошо до этого момента.

    x + 2 <0 или x - 3 <0 НЕПРАВИЛЬНО!

    Если произведение двух чисел равно , равно 0, то хотя бы одно из чисел должно быть 0. Однако произведение двух отрицательных чисел не является отрицательным, поэтому этот подход бесполезен для решения неравенств.

    Пример 9 .

    1.2 x 3 + 3,07 x 2 — x — 3,71> 0.

    Эта задача намного сложнее неравенства в предыдущем примере! Фактор непросто, поэтому мы не сможем найти точные значения критических чисел. Мы будем использовать графическую утилиту, чтобы приблизить критические числа. График полинома показан ниже.

    y = 1,2 x 3 + 3,07 x 2 — x — 3,71

    Критическое число составляет примерно -2.35, -1,25 и 1,05. В этой задаче мы ищем регионы, где график находится над осью.

    Набор решений : (-2.35, -1.25) union (1.05, inf).

    Упражнение 4 :

    Решите неравенство x 2 + 3x — 4> 0. Используйте графическую утилиту, чтобы проверить свое решение.

    Вернуться к содержанию

    Рациональные неравенства

    Рациональное выражение — это полином, деленный на полином.В общем, графики рациональных функций есть перерывы. Они не определены в нулях знаменателя. Это единственные места, где есть ломается, поэтому мы можем использовать ту же технику для решения рациональных неравенств, которую мы используем для полиномиальных неравенств.

    Пример 10 .

    Критические числа для рационального неравенства — это все нули числителя и знаменателя. С числитель и знаменатель уже учтены в этом примере, мы видим, что критические числа равны -3, 5 и 1.

    Три критических числа делят числовую строку на четыре тестовых интервала.

    (-inf, -3): -4 находится в интервале, а рациональная функция, оцененная как -4, равна -9/15. Поскольку значение отрицательное, график рациональной функции находится на ниже оси x на всем интервале.

    (-3, 1): 0 находится в интервале. Значение функции в 0 равно 5, что положительно. График функции находится на выше по оси x во всем интервале.

    (1, 5): 2 находится в интервале. Значение 2 равно -5. График функции находится на ниже по оси абсцисс.

    (5, inf): 6 находится в интервале. Значение 6 — 9/15. График функции находится на выше по оси абсцисс.

    Мы ищем области, в которых график находится выше оси x, поэтому набор решений равен (-3, 1) union (5, инф.).

    Примечание: Можно использовать графическую утилиту, чтобы увидеть, с какой стороны оси x находится график над различные интервалы тестирования.В некоторых случаях вам необходимо решить алгебраически, чтобы найти точные значения критических чисел, но как только это будет сделано, график предоставит быстрый способ решить проблему.

    График y = (x + 3) (x — 5) / 3 (x — 1)

    При работе с неравенством следует помнить о двух важных моментах:

    1.

    | x |

    | x | > a тогда и только тогда, когда x <-a или x> a.

Добавить комментарий

Ваш адрес email не будет опубликован. Обязательные поля помечены *